SlideShare una empresa de Scribd logo
1 de 22
A 27-year-old sexually active male develops a vesiculobullous lesion on the glans soon after
taking tablet paracetamol for fever. The lesion healed with hyperpigmentation. The most
likely diagnosis is:
A: Behcet's syndrome
B:
Herpes
genitalis
C: Fixed drug eruption
D: Pemphigus vulgaris
Correct Ans:C
Explanation
An adverse cutaneous reaction to an ingested drug with characteristic clinical features is
fixed drug eruption. Presentation with a solitary lesion in the genital skin that heals with
hyperpigmentation. Soon after ingestion of paracetamol, one of the most commonly
implicated agents for FDE, leads to the diagnosis of FDE.
Ref: Anthony Du Vivier, Phillip H. McKee, Chapter 17, “Reactive Disorder of The Skin and
Drug Eruptions”, In the book, “Atlas of Clinical Dermatology”, Elsivier Publication, 2002,
3rd Edition, Spain, Page 367
Sample Previous Year Question on Pemphigus vulgaris based on previous Year
Questions of NEET PG, USMLE,PLAB,FMGE (MCI Screening). Please visit
www.medicoapps.org for more such Quizzes
A 22 year old female presents with dirty warty papules in the seborrheic areas especially in
the summer. Her father also gives a h/o similar lesions. What is the most probable
diagnosis?
A: Pemphigus foliaceus
B:
Keratosis
pilaris
C: Darier’s disease
D: Seborrheic dermatitis
Correct Ans:C
Explanation
It is also known as Keratosis follicularis with autosomal dominant inheritance.
Sample Previous Year Question on Pemphigus vulgaris based on previous Year
Questions of NEET PG, USMLE,PLAB,FMGE (MCI Screening). Please visit
www.medicoapps.org for more such Quizzes
The “dilapidated brick wall” appearance is classically seen in the histopathology of the
following?
A: Hailey Hailey disease
B:
Pemphigus
vulgaris
C: Darier's disease
D: Bullous pemphigoid
Correct Ans:A
Explanation
Hailey Hailey disease/familial benign chronic pemphigus has full thickness partial
acantholysis resulting in this appearance.
Ref: Rook Textbook of Dermatology, 8th Edition, Page 39.33.
Sample Previous Year Question on Pemphigus vulgaris based on previous Year
Questions of NEET PG, USMLE,PLAB,FMGE (MCI Screening). Please visit
www.medicoapps.org for more such Quizzes
Koebner's phenomenon is seen in all of the following conditions, EXCEPT:
A: Lichen planus
B:
Ichthyosi
s
C: Psoriasis
D: Pemphigus
Correct Ans:B
Explanation
Koebner’s phenomenon refers to development of morphologically identical lesions in the
traumatized uninvolved skin of the patients who have cutaneous diseases. It is also known
as isomorphic phenomenon.
Conditions associated with Koebner’s phenomenon are:
 Warts
 Molluscum contagiosum
 Autoimmune (vitiligo)
 Psoriasis
 Pemphigus
 Lichen planus
Ref: An Aid to the MRCP: Essential Lists, Facts and Mnemonics By Nicholas Boeckx, page
184.
Sample Previous Year Question on Pemphigus vulgaris based on previous Year
Questions of NEET PG, USMLE,PLAB,FMGE (MCI Screening). Please visit
www.medicoapps.org for more such Quizzes
A 30 year old pregnant woman presents to a physician with painful oral ulcers. Physical
examination demonstrates widespread erosions of her mucous membranes. Close
examination reveals a friable mucosa, but no well-defined aphthous ulcers. Biopsy of
perilesional mucosa demonstrates acantholysis; direct immunofluorescence demonstrates
an intraepidermal band of IgG and C3. Which of the following is the most likely diagnosis?
A: Bullous pemphigoid
B: Dermatitis herpetiformis
C: Herpes simplex I
D: Pemphigus vulgaris
Correct Ans:D
Explanation
This is pemphigus vulgaris, in which autoantibody directed against transmembrane
cadherin adhesion molecules induced acantholysis (breakdown of epithelial cell-cell
connections) with resulting intraepidermal blister formation. It may develop spontaneously
or following triggers such as drugs (thiols, penicillamine), physical injury (burns), cancer,
pregnancy, other skin diseases, and emotional stress. Pemphigus vulgaris is a relatively
rare blistering disease; it is seen more commonly in patients with Jewish or Mediterranean
heritage. In addition to the usually prominent oral ulcers, uncomfortable skin erosions can
also occur when the blisters rupture rapidly and are not observed. The epidermis at the
edge of these erosions is often easily disrupted by sliding pressure (Nikolsky sign).
Bullous pemphigoid is characterized by deeper blisters, occurring at the dermal-epidermal
junction.
Dermatitis herpetiformis is characterized by severe, intense pruritus and groups of papules
and vesicles.
Herpes simplex I or II can show multinucleated giant cells on scrapings of the ulcer base.
Ref: Suurmond D. (2009). Section 34. Disorders of the Mouth. In D. Suurmond
(Ed), Fitzpatrick's Color Atlas & Synopsis of Clinical Dermatology, 6e.
Sample Previous Year Question on Pemphigus vulgaris based on previous Year
Questions of NEET PG, USMLE,PLAB,FMGE (MCI Screening). Please visit
www.medicoapps.org for more such Quizzes
Tzanck preparation is used for the following skin conditions, EXCEPT:
A: Pemphigus
B: SSS syndrome
C: Senear Usher syndrome
D: Fungal infections
Correct Ans:D
Explanation
Microscopic examination of cells obtained from the base of vesicles (Tzanck preparation)
may reveal the presence of acantholytic cells in the acantholytic diseases (e.g., pemphigus
or SSS syndrome) or of giant epithelial cells and multinucleated giant cells (containing 10–
12 nuclei) in herpes simplex, herpes zoster, and varicella. Material from the base of a
vesicle obtained by gentle curettage with a scalpel is smeared on a glass slide, stained with
either Giemsa or Wright stain or methylene blue, and examined to determine whether there
are acantholytic or giant epithelial cells, which are diagnostic.
Pemphigus erythematosus is also known as Senear-Usher syndrome.
Ref: (2013). Introduction. In Wolff K, Johnson R, Saavedra A.P. (Eds), Fitzpatrick’s Color
Atlas and Synopsis of Clinical Dermatology, 7e.
Sample Previous Year Question on Pemphigus vulgaris based on previous Year
Questions of NEET PG, USMLE,PLAB,FMGE (MCI Screening). Please visit
www.medicoapps.org for more such Quizzes
A 30 year old male presents with itchy papulo-vesicular lesions on the extremities, knees,
elbows and buttocks since one year. Direct immunofluorescence staining of the lesions
showed IgA deposition at dermoepidermal junction. Which of the following represents the
most probable diagnosis?
A:
Pemphigus
vulgaris
B: Bullous pemphigoid
C: Dermatitis herpetiformis
D: Nummular eczema
Correct Ans:C
Explanation
Patient is showing features of dermatitis herpetiformis. Dermatitis herpetiformis is a chronic, intensely
itchy blistering disease characterized by the presence of papules and vesicles occurring predominantly
on the extensor surfaces of the body such as the elbows, knees, buttocks and scapula. It is seen in
association with gluten sensitive enteropathy.
Diagnosis is made by the presence of IgA deposits in the dermoepidermal junction by direct
immunoflouresence examination of a perilesional or normal appearing skin biopsy sample. Treatment
includes dapsone and gluten free diet.
Ref: NORD Guide to Rare Disorders: National Organization for Rare Disorders, Page 103; Atlas of
Clinical Dermatology By Anthony Du Vivier, 3rd Edition, Pages 418, 426; Dermatology in
Clinical Practice By S. W. Lanigan, Page 170
Sample Previous Year Question on Pemphigus vulgaris based on previous Year
Questions of NEET PG, USMLE,PLAB,FMGE (MCI Screening). Please visit
www.medicoapps.org for more such Quizzes
A 40 year old male developed persistant oral ulcers followed by multiple flaccid bullae on
trunk and extremities. Direct immunofluorescence examination of a skin biopsy showed
intercellular IgG deposits in the epidermis. The most probable diagnosis is:
A:
Pemphigus
vulgaris
B: Bullous Pemphigoid
C: Bullous Lupus erythematosus
D: Epidermolysis bullosa acquisita
Correct Ans:A
Explanation
It is an autoimmune intraepidermal blistering disease due to antibodies against desmoglein
3. It begins as painful oral erosions followed by flaccid blisters on the trunk, face, scalp.
Ref: Current Literature Dermatology By Pasricha, Pages 197, 202; Rook’s Textbook of
Dermatology, 8th Edition.
Sample Previous Year Question on Pemphigus vulgaris based on previous Year
Questions of NEET PG, USMLE,PLAB,FMGE (MCI Screening). Please visit
www.medicoapps.org for more such Quizzes
Which of the following disorder is associated with acantholysis?
A: Pemphigoid
B: Pemphigus vulgaris
C: Erythema multiforme
D: Dermatitis herpetiformis
Correct Ans:B
Explanation
Acantholysis refers to loss of cohesion between epidermal cells as a result of destruction of
intercellular substance. Acantholytic cells are round cells with large hyperchromatic nuclei
and perinuclear halo. The primary lesion of pemphigus vulgaris is acantholysis in the
epidermis. Its histologic hallmark is acantholysis which is suprabasal, and the basal cells
remain attached to the basement membrane.
Ref: Tropical Dermatopathology By Singh, Page 121; Pathology of The Head and Neck By
Antonio Cardesa, Page 74.
Sample Previous Year Question on Pemphigus vulgaris based on previous Year
Questions of NEET PG, USMLE,PLAB,FMGE (MCI Screening). Please visit
www.medicoapps.org for more such Quizzes
A 45 yr old female developed recurrent oral erosions followed by multiple flaccid bullae on
trunk and extremities. A tzanck smear showed acantholytic cells and direct
immunofluoroscence showed intercellular IgG deposits in the epidermis. The most probable
diagnosis is:
A: Bullous Pemphigoid
B: Stevens Johnson syndrome
C: Herpes simplex 1 infection
D: Pemphigus vulgaris
Correct Ans:D
Explanation
Recurrent, painful oral lesions and cutaneous lesions in the form of flaccid bullae are
characteristic of phemphigus vulgaris. Positive Tzanck smear, IgG deposits on direct
immunofluoroscence positive nikolsky’s sign are features of phemphigus vulgaris.
Ref: Textbook of Dermatology, Venereology and Leprology By Devinder Mohan Thappa, 2nd
Edition, Chapter 12, Pages 152-53; Dermatology (A coloured Hand Textbook) By R. J. G.
Rycroft, S. J. Robertson, Sarah H. Wakeli, 2nd Edition, Part 1, Pages 54, 55
Sample Previous Year Question on Pemphigus vulgaris based on previous Year
Questions of NEET PG, USMLE,PLAB,FMGE (MCI Screening). Please visit
www.medicoapps.org for more such Quizzes
Itching associated with linear IgA deposition in dermal papillae is a feature of:
A: Bullous disease of childhood
B:
Lichenoid bullous
disease
C: Dermatitis herpetiformis
D: Pemphigus vulgaris
Correct Ans:C
Explanation
The cutaneous lesions of Dermatitis herpetiformis (DH) in childhood resemble those in adult and are
severely pruritic papulovesicles. DH is associated with granular deposition of IgA antibody within the
dermal papillae.
Ref: Cambridge Textbook of Accident and Emergency Medicine edited by David V. Skinner, Andrew
Swain, Colin Robertson, J. W. Rodney Peyton, 1997, Page 1051
Sample Previous Year Question on Pemphigus vulgaris based on previous Year
Questions of NEET PG, USMLE,PLAB,FMGE (MCI Screening). Please visit
www.medicoapps.org for more such Quizzes
Which of the following drug is not associated with drug induced pemphigus?
A: Rifampicin
B:
Penicilli
n
C: Captopril
D: Furosemide
Correct Ans:D
Explanation
Penicillin, cephalosporins, penicillamine, rifampicin, captopril, piroxicam, phenylbutazone
are drugs commonly associated with drug induced pemphigus.
Sample Previous Year Question on Pemphigus vulgaris based on previous Year
Questions of NEET PG, USMLE,PLAB,FMGE (MCI Screening). Please visit
www.medicoapps.org for more such Quizzes
Pemphigus vulgaris is characterized by all, except:
A: Positive Nikolsky’s sign
B:
Oral
erosions
C: Subepidermal bulla
D: Tzanck smear showing acantholytic cells
Correct Ans:C
Explanation
Pemphigus vulgaris is an intraepidermal blistering disease.
Sample Previous Year Question on Pemphigus vulgaris based on previous Year
Questions of NEET PG, USMLE,PLAB,FMGE (MCI Screening). Please visit
www.medicoapps.org for more such Quizzes
Scenario: A 35 year old lady is diagnosed with Pemphigus Vulgaris by her dermatologist.
Assertion: Pemphigus vulgaris presents with flaccid blisters and oral erosions in a middle aged
person.
Reason: It is a intraepidermal blistering disorder due to antibodies against desmogleins.
A: Both Assertion and Reason are true, and Reason is the correct explanation for Assertion
B:
Both Assertion and Reason are true, and Reason is not the
correct explanation for Assertion
C: Assertion is true, but Reason is false
D: Assertion is false, but Reason is true
Correct Ans:A
Explanation
Pemphigus vulgaris is an intraepidermal blistering disease due to antibodies against
desmogleins (epidermal attachment complexes). It presents with flaccid blisters and oral
erosions in a middle aged person. The subepidermal blistering disorders present with tense
blisters.
Ref: Rook’s Textbook of Dermatology, 8th Edition, Page 40.3
Sample Previous Year Question on Pemphigus vulgaris based on previous Year
Questions of NEET PG, USMLE,PLAB,FMGE (MCI Screening). Please visit
www.medicoapps.org for more such Quizzes
A person presents with hemorrhagic fluid in a tense blister at dermoepidermal junction.
Most probable diagnosis is:
A: Pemphigoid
B: Pemphigus vulgaris
C: Pemphigus vegetans
D: Drug induced pemphigus
Correct Ans:A
Explanation
The pemphigoid group includes a series of chronic diseases in which blisters form within
the lamina lucida of the basement membrane dermoepidermal junction). The blister roof is
thus the entire epidermis so it is stable. Frequently the blisters are hemorrhagic, fluid filled
and tense. No accantholysis occurs, so the Tzanck test is negative. Antibodies are directed
against various components of the basement membrane zone.
Characteristic of pemphigus vulgaris is an intradermal blister occurring immediately above
the basal cell layer and associated with acantholytic cells. The blisters rapidly ruptures,
leaving non healing, painful erosions.
Pemphigus vegetans usually presents clinically as serpiginous ulcers. Microscopically, the
epithelium tends to proliferate and become verruciform. Acantholytic cells may not be
conspicuous and eosinophil microabscesses are the most typical histological feature.
Drug induced pemphigus can present as pemphigus foliaceous, pemphigus erythematosus,
or as pemphigus vulgaris. Moist erythematous crusted scaling plaques and superficial
friable blisters are seen. Common drugs responsible are thiol compounds, drugs having a
sulph-hydryl group, penicillamine.
Sample Previous Year Question on Pemphigus vulgaris based on previous Year
Questions of NEET PG, USMLE,PLAB,FMGE (MCI Screening). Please visit
www.medicoapps.org for more such Quizzes
Fogo selvagem is a type of:
A:
Pemphigus
vulgaris
B: Pemghigus vegetans
C: Pemphigus foliaceus
D:
Bullous
pemphigoid
Correct Ans:C
Explanation
Endemic pemphigus foliaceus common in rural parts of South America, particularly certain
states of Brazil is known as fogo selvagem (wild fire) caused by bite of black fly.
Ref: Rook's textbook of dermatology, 8th edition, Pg 40.13
Sample Previous Year Question on Pemphigus vulgaris based on previous Year
Questions of NEET PG, USMLE,PLAB,FMGE (MCI Screening). Please visit
www.medicoapps.org for more such Quizzes
FTU is a measure of:
A: Area involved in severe drug reactions
B:
Amount of topical drug to be
applied
C: Mucosal involvement in pemphigus
D: Concentration of drug in topical preparations
Correct Ans:B
Explanation
An approximate practical measure of topical medication is the fingertip unit, the quantity of
ointment, extruded from a tube with a nozzle of 5 mm diameter extending from the distal
crease of the forefinger to ventral aspect of the fingertip. This unit weighs approximately
0.49 g in males and 0.43 g in females and covers, on average, an area of approx 300 cm2.
Sample Previous Year Question on Pemphigus vulgaris based on previous Year
Questions of NEET PG, USMLE,PLAB,FMGE (MCI Screening). Please visit
www.medicoapps.org for more such Quizzes
Which of the following presents with a "string of pearl appearance"?
A:
Pemphigus
vulgaris
B: Bullous pemphigoid
C: Linear IgA disease
D: Dermatitis herpetiformis
Correct Ans:C
Explanation
The lesions in Linear IgA disease with a bimodal age distribution ie <5yrs & > 60 yrs
comprise of urticated plaques and papules, and annular, polycyclic lesions often with
blistering around the edge, the string of pearls sign/cluster of jewels sign.
Sample Previous Year Question on Pemphigus vulgaris based on previous Year
Questions of NEET PG, USMLE,PLAB,FMGE (MCI Screening). Please visit
www.medicoapps.org for more such Quizzes
A biopsy of affected skin in Pemphigus vulgaris would show which of the following?
A: Acantholysis
B: Balloon degeneration
C: Reticular changes
D: Spongiosis
Correct Ans:A
Explanation
The pathologic changes in pemphigus vulgaris are acantholysis, clefts, and blister formation
in the intraepidermal areas. Acantholysis is the separation of keratinocytes from one
another. The loss of cohesion or contact between cells begins with the detachment of
tonofilaments from desmosomes. Evidence indicates that an IgG autoantibody actually
induces these changes.
Good to know:
 Balloon degeneration and reticular changes are both histologic markers
of herpes simplex viral-induced lesions.
 Spongiosis is a general term referring to serous exudates between cells of
the epidermis, with an inflammatory infiltrate in the dermis. This is
frequently seen indermatitis lesions.
Ref:Longo D.L., Fauci A.S., Kasper D.L., Hauser S.L., Jameson J.L., Loscalzo J. (2012).
Chapter 54. Immunologically Mediated Skin Diseases. In D.L. Longo, A.S. Fauci, D.L. Kasper,
S.L. Hauser, J.L. Jameson, J. Loscalzo (Eds), Harrison's Principles of Internal Medicine, 18e.
Sample Previous Year Question on Pemphigus vulgaris based on previous Year
Questions of NEET PG, USMLE,PLAB,FMGE (MCI Screening). Please visit
www.medicoapps.org for more such Quizzes
Which of the following tests would be MOST useful for confirmation of Pemphigus vulgaris?
A: Bacterial culture
B: Complete blood count
C: Fungal culture
D: Tissue biopsy with direct immunofluorescence
Correct Ans:D
Explanation
Direct immunofluorescence (DIF) is of great value in the early diagnosis of pemphigus
vulgaris. DIF shows intercellular IgG throughout the epidermis or the oral epithelium. IgG
is found in both involved and clinically normal skin in nearly all patients with pemphigus. In
acantholytic areas, C3 deposition is also reliably found. DIF results remain positive for a
long time and may still be positive many years after clinical remission.
 Bacterial culture is helpful in superinfected lesions, but does not help in
confirming a diagnosis of pemphigus.
 Complete blood count is helpful in infectious processes.
 Fungal culture is used in tinea lesions.
Ref: Longo D.L., Fauci A.S., Kasper D.L., Hauser S.L., Jameson J.L., Loscalzo J. (2012).
Chapter 54. Immunologically Mediated Skin Diseases. In D.L. Longo, A.S. Fauci, D.L. Kasper,
S.L. Hauser, J.L. Jameson, J. Loscalzo (Eds), Harrison's Principles of Internal Medicine, 18e.
Sample Previous Year Question on Pemphigus vulgaris based on previous Year
Questions of NEET PG, USMLE,PLAB,FMGE (MCI Screening). Please visit
www.medicoapps.org for more such Quizzes
Patient with gluten-sensitive enteropathy has a lifelong history of periodic crops of
intensely pruritic, grouped, papular or vesicular lesions on the elbows, knees, sacrum, and
shoulders. Because the vesicles are intensely pruritic, the patient routinely scratches the
top off them, which relieves the pruritus. Which of the following is the most likely
diagnosis?
A: Bullous pemphigoid
B: Dermatitis herpetiformis
C: Herpes simplex I
D: Pemphigus vulgaris
Correct Ans:B
Explanation
The condition described is dermatitis herpetiformis, which is strongly associated with
gluten-sensitive enteropathy (celiac sprue), and often has a life-long, intermittent course.
Clinically, patients have (excoriated) groups of papules and vesicles on an erythematous
base. The lesions tend to involve the extensor surfaces of the extremities and the buttocks.
Microscopically, the lesions show subepidermal papillary dermal neutrophilic abscesses,
with granular deposits of IgA and C3 in dermal papillary tips. Patients may respond
dramatically to dapsone therapy.
Bullous pemphigoid produces large, tense blisters.
Herpes simplex I is characterized by crops of vesicles, but is not particularly associated
with gluten-sensitive enteropathy.
Pemphigus vulgaris usually shows prominent oral involvement.
Ref: DeWitt C.A., Buescher L.S., Stone S.P. (2012). Chapter 153. Cutaneous Manifestations
of Internal Malignant Disease: Cutaneous Paraneoplastic Syndromes. In L.A. Goldsmith, S.I.
Katz, B.A. Gilchrest, A.S. Paller, D.J. Leffell, N.A. Dallas (Eds), Fitzpatrick's Dermatology in
General Medicine, 8e.
Sample Previous Year Question on Pemphigus vulgaris based on previous Year
Questions of NEET PG, USMLE,PLAB,FMGE (MCI Screening). Please visit
www.medicoapps.org for more such Quizzes
'Bulla spread sign' is seen in which of the following condition?
A: Herpes gestationalis
B: Bullous pemphigoid
C: Pemphigus vulgaris
D: Herpes simplex
Correct Ans:C
Explanation
Pemphigus vulgaris is associated with positive Nikolsky sign and Asboe- Hansen sign or
Bullae spread sign. In positive bullae spread sign, pressing the top of existing bullae will
lead to spread of bullae to adjacent normal appearing skin. Positive Nikolsky sign refers to
shearing of the epidermis by exerting lateral pressure on the normal skin adjacent to an
active blister.
Pemphigus vulgaris is an acquired chronic disease in which blisters develop on normal
appearing skin and mucous membrane. In this condition the pemphigus antibodies attach
to extracellular domain of desmoglein and interfere with its attachment to similar domains
on other cells reducing cell cell adherence. It is characterised by the development of flaccid
clear blister which rupture and evolve into an inflamed erosion.
Ref: Blistering Skin Diseases By Lawrence S. Chan page 26.
Sample Previous Year Question on Pemphigus vulgaris based on previous Year
Questions of NEET PG, USMLE,PLAB,FMGE (MCI Screening). Please visit
www.medicoapps.org for more such Quizzes
A 30 year old male presented with itchy, papulovesicular lesions in the extensor aspect of
elbows and knees. Biopsy of the lesion showed dermoepidermal blister with micro-
abscesses. Immunofluorescence showed deposits of IgA in the tips of dermal papillae. What
is the MOST likely diagnosis?
A: Bullous Impetigo
B: Bullous pemphigoid
C: Pemphigus vulgaris
D: Dermatitis herpetiformis
Correct Ans:D
Explanation
sensitive enteropathy. In this condition absorption of gluten induces the formation of
circulating immune complexes which deposit in the dermal papillae causing inflammation
and dermoepidermal split.
Skin lesions are in the form of extremely itchy grouped papulovesicular lesions that develop
on normal or erythematous skin. The most frequently involve the extensors(elbow, knee,
buttocks, shoulder and sacral area). Biopsy shows: dermoepidermal blister with collection
of polymorphs at the tip of dermal papillae.Immunofluorescence showed deposits of IgA in
the tips of dermal papillae. Serology: Anti endomysial and anti reticulin antibodies are
present in most patients.
Ref: Illustrated Synopsis Of Dermatology & Sexually Transmitted Diseases (3Nd By Khanna
page 71.
Sample Previous Year Question on Pemphigus vulgaris based on previous Year
Questions of NEET PG, USMLE,PLAB,FMGE (MCI Screening). Please visit
www.medicoapps.org for more such Quizzes
Intraepidermal bullae formation occur in which of the following condition?
A: Bullous Impetigo
B: Bullous pemphigoid
C: Pemphigus vulgaris
D: Dermatitis herpetiformis
Correct Ans:C
Explanation
Pemphigus vulgaris is an intraepidermal bullous disorder where the split is suprabasal.
Pemphigus is an autoimmune disorder characterised by the production of IgG
autoantibodies against desmoglein which are involved in intercellular adhesion. These
antibodies induce keratinocytes to release serine proteases which dissolve the intercellular
substance.
The site of split in 4 main variants of pemphigus are:
 Pemphigus vulgaris: split is suprabasal
 Pemphigus vegetans: split is suprabasal
 Pemphigus foliaceus: blister is either in the granular layer or just below the
horny layer
 Pemphigus erythematosus: blister is either in the granular layer or just
below stratum corneum
Ref: Illustrated Synopsis Of Dermatology & Sexually Transmitted Diseases By Khanna page
64.
Sample Previous Year Question on Pemphigus vulgaris based on previous Year
Questions of NEET PG, USMLE,PLAB,FMGE (MCI Screening). Please visit
www.medicoapps.org for more such Quizzes
A 30 year old male presents to the dermatologist with silvery, scaling plaques on his elbows
and knees. His mother has been afflicted with the same condition in the past. The most
likely diagnosis is ?
A:
Acne
rosacea
B: Acne vulgaris
C: Pemphigus vulgaris
D: Psoriasis vulgaris
Correct Ans:D
Explanation
Psoriasis vulgaris usually appears on the nails, knees, elbows, and scalp. It does not
generally affect the mucous membranes. Lesions are well-demarcated coral-colored
plaques with a white or silvery scale (classic clue). Histologically, epidermal hyperplasia
causing thickening and lengthening of the rete ridges is apparent, as is thinning of the
epidermis over the dermal papillae. There is a recognized genetic component to this
condition. Peak incidence is at age 30.
Here's a point-saving strategy: It is easy to get confused on this question because of the
similar looking answer choices, especially under time pressure. Be extra careful in such
cases to mark the proper choice on your answer grid.
Acne rosacea affects the central face. Erythema, telangiectasias, acneform lesions (papules,
cysts, pustules), and rhinophyma (telangiectasias and hyperplasia of nasal soft tissue) are
found in various combinations. It is common from ages 30-50. Women are affected three
times more frequently than men, but the syndrome is more severe in the latter.
Acne vulgaris causes comedones, papules, and cysts. It may be related to hormones, drugs,
diet, irritants, and genetic factors. Allergy to Propionibacterium acnes has been found to
contribute to this condition.
Pemphigus vulgaris starts with small vesicles, usually on the oral or nasal mucosa, then
spreads to other parts of the body. Bullae are delicate and flaccid. The condition is due to
autoantibodies to intercellular junctions between keratinocytes. Nikolsky's sign (production
of blistering by light stroking or rubbing of the skin) is positive. Pemphigus is most common
from ages 40-60.
Ref: Suurmond D. (2009). Section 3. Psoriasis. In D. Suurmond (Ed), Fitzpatrick's Color
Atlas & Synopsis of Clinical Dermatology, 6e.
Sample Previous Year Question on Pemphigus vulgaris based on previous Year
Questions of NEET PG, USMLE,PLAB,FMGE (MCI Screening). Please visit
www.medicoapps.org for more such Quizzes
A 60 year old male presents to the dermatologist with a complaint of tender blisters on his
arm and flank. Physical examination reveals blisters and flaccid bullae; a few have
ruptured, leaving red, sore, denuded areas. Which of the following findings would suggest
the diagnosis of pemphigus vulgaris as opposed to bullous pemphigoid?
A: Eosinophils within bullae
B: IgA deposits on basement membrane
C: Negative Nikolsky sign
D: Oral mucosal lesions
Correct Ans:D
Explanation
Pemphigus vulgaris is associated with chronic, severe bullae formation on the skin and oral
mucosa. Mucosal lesions are extremely rare in bullous pemphigoid. This can be used
clinically to guide therapy, although a skin biopsy should be taken to confirm the diagnosis.
Both diseases are characterized by formation of tender bullae that can rupture,leaving red,
raw areas. Pemphigus vulgaris patients eventually became febrile and lose weight, and if
untreated, most will die within one year. Bullous pemphigoid lesions tend to heal and the
patients do very well. This prognostic difference is an important distinction in
dermatological medicine.
Eosinophils within blisters provide an important clue supporting bullous pemphigoid as the
diagnosis which must be ascertained with histologic examination. The vesicles in
pemphigus vulgaris mostly contain rounded acantholytic keratinocytes "floating" within.
IgA deposits seen with immunofluorescence on the basement membrane is characteristic of
a third bullae-forming disease known as dermatitis herpetiformis. Vesicles form between
the dermal-epidermal junction. The disease is associated with gluten sensitivity and celiac
disease.
IgG autoantibody activity is common to both pemphigus vulgaris and bullous pemphigoid.
Ref: Payne A.S., Stanley J.R. (2012). Chapter 54. Pemphigus. In K. Wolff (Ed),Fitzpatrick's
Dermatology in General Medicine, 8e.
Sample Previous Year Question on Pemphigus vulgaris based on previous Year
Questions of NEET PG, USMLE,PLAB,FMGE (MCI Screening). Please visit
www.medicoapps.org for more such Quizzes
The day after playing in the high grasses of a neighbor's fields, an 8 year old boy develops a
weeping, vesicular, erythematous, and itchy rash on his arms, legs, and neck. Which of the
following best describes this inflammatory response?
A: Erythema nodosum
B:
Pemphigu
s
C: Psoriasis
D: Spongiotic dermatitis
Correct Ans:D
Explanation
Spongiotic dermatitis is seen in cases of contact dermatitis, such as poison ivy exposure.
The accumulation of inflammatory cells in the superficial dermis causes marked edema,
which splays epidermal keratinocytes apart and giving a spongy appearance to intercellular
bridges. Grossly, the skin has a weepy appearance with frequent blistering.
Erythema nodosum is a form of panniculitis, which is chronic inflammation in the
subcutaneous fat lobules. Erythema nodosum presents as painful erythematous nodules,
often with fever and malaise. It is associated with infections and drug reactions, and is not
a contact dermatitis.
Pemphigus is a genetic blistering disorder due to the production of antibodies to the
intercellular cement substances in skin and mucous membranes.
Psoriasis is a common chronic inflammatory disease causing plaques and scales, typically
on elbows, knees, and scalp. The pathogenesis of psoriasis is still unclear; it may be a
complement-mediated autoimmune process.
Ref: McCalmont T.H. (2010). Chapter 8. Diseases of the Skin. In S.J. McPhee, G.D. Hammer
(Eds), Pathophysiology of Disease, 6e.
Sample Previous Year Question on Pemphigus vulgaris based on previous Year
Questions of NEET PG, USMLE,PLAB,FMGE (MCI Screening). Please visit
www.medicoapps.org for more such Quizzes
A patient presents with scarring Alopecia, thinned nails, hypopigmented macular lesions
over trunk and oral mucosa. The diagnosis is :
A: Psoriasis
B: Lepros
y
C: Lichen planus
D: Pemphigus
Correct Ans:C
Explanation
Lichen planus (LP) is a papulosquamous disorder that may affect the skin, scalp, nails, and
mucous membranes. The primary cutaneous lesions are pruritic, polygonal, flat-topped,
violaceous papules. Close examination of the surface of these papules often reveals a
network of gray lines (Wickham's striae). The skin lesions may occur anywhere but have a
predilection for the wrists, shins, lower back, and genitalia. Involvement of the scalp, lichen
planopilaris, may lead to scarring alopecia, and nail involvement may lead to permanent
deformity or loss of fingernails and toenails. LP commonly involves mucous membranes,
particularly the buccal mucosa, where it can present a spectrum of disease from a mild,
white, reticulate eruption of the mucosa, to a severe, erosive stomatitis. Erosive stomatitis
may persist for years and may be linked to an increased risk of oral squamous cell
carcinoma.
Ref: Harrison's Internal Medicine;Part 2. Cardinal Manifestations and Presentation of
Diseases ;Section 9. Alterations in the Skin ;Chapter 53. Eczema, Psoriasis, Cutaneous
Infections, Acne, and Other Common Skin Disorders-Eczema and Dermatitis
Sample Previous Year Question on Pemphigus vulgaris based on previous Year
Questions of NEET PG, USMLE,PLAB,FMGE (MCI Screening). Please visit
www.medicoapps.org for more such Quizzes
A patient presents with scarring Alopecia, thinned nails, hypopigmented macular lesions
over trunk and oral mucosa. The diagnosis is:
A: Psoriasis
B:
Lepros
y
C: Lichen planus
D: Pemphigus
Correct Ans:C
Explanation
Lichen planus (LP) is a papulosquamous disorder that may affect the skin, scalp, nails, and
mucous membranes. The primary cutaneous lesions are pruritic, polygonal, flat-topped,
violaceous papules. Close examination of the surface of these papules often reveals a
network of gray lines (Wickham's striae). The skin lesions may occur anywhere but have a
predilection for the wrists, shins, lower back, and genitalia. Involvement of the scalp, lichen
planopilaris, may lead to scarring alopecia, and nail involvement may lead to permanent
deformity or loss of fingernails and toenails. LP commonly involves mucous membranes,
particularly the buccal mucosa, where it can present a spectrum of disease from a mild,
white, reticulate eruption of the mucosa, to a severe, erosive stomatitis. Erosive stomatitis
may persist for years and may be linked to an increased risk of oral squamous cell
carcinoma.
Ref: Harrison's Internal Medicine;Part 2. Cardinal Manifestations and Presentation of
Diseases ;Section 9. Alterations in the Skin ;Chapter 53. Eczema, Psoriasis, Cutaneous
Infections, Acne, and Other Common Skin Disorders-Eczema and Dermatitis.
Sample Previous Year Question on Pemphigus vulgaris based on previous Year
Questions of NEET PG, USMLE,PLAB,FMGE (MCI Screening). Please visit
www.medicoapps.org for more such Quizzes
'Bulla spread sign' is seen in :
A: Herpes gestational
B: Bullous pemphigoid
C: Pemphigus vulgaris
D: Herpes simplex
Correct Ans:C
Explanation
In the traditional "bulla spread" sign or Lutz sign, the margin of an intact bullae is first
marked by a pen. Slow, careful and unidirectional pressure applied by a finger to the bulla
causes peripheral extension of the bulla beyond the marked margin. The bulla thus
extended has an irregular angulated border in pemphigus vulgaris, while a regular rounded
border is observed in bullous pemphigoid or other subepidermal blistering disorders.
Ref :Longo D.L., Fauci A.S., Kasper D.L., Hauser S.L., Jameson J.L., Loscalzo J. (2012).
Chapter 54. Immunologically Mediated Skin Diseases. In D.L. Longo, A.S. Fauci, D.L. Kasper,
S.L. Hauser, J.L. Jameson, J. Loscalzo (Eds), Harrison's Principles of Internal Medicine, 18e.
Sample Previous Year Question on Pemphigus vulgaris based on previous Year
Questions of NEET PG, USMLE,PLAB,FMGE (MCI Screening). Please visit
www.medicoapps.org for more such Quizzes
Berkley membrane is seen in the following cutaneous condition:
A: Psoriasis
B: Pemphigus
C: Tinea capitis
D: Pityriasis rubra
Correct Ans:A
Explanation
Psoriasis is characterized by well-circumscribed, sharply demarcated erythematous papules
and/or plaques. These are covered by dry, brittle, silvery or grayish white, loosely
adherent, micaceous scales. The scales are disposed in lamellar fashion. Occasionally a
white blanching ring is seen around the psoriatic lesions, known asWoronoff ring. On
grattage, silvery white scales come off in layers. After their removal, a characteristic
coherence is observed, as if one scratches on a wax candle (candle grease sign). On further
grattage, a thin peel like membrane, Berkley’s membrane, is seen which comes off as a
whole. On its removal, a wet surface with multiple pinpoint bleeding is revealed. This is
called Auspitz sign.
Ref: Textbook of Clinical Dermatology By Sehgal, 4th edition, Page 127.
Sample Previous Year Question on Pemphigus vulgaris based on previous Year
Questions of NEET PG, USMLE,PLAB,FMGE (MCI Screening). Please visit
www.medicoapps.org for more such Quizzes
Nikolsky's sign is positive in each of the following conditions, EXCEPT:
A: Pemphigus
B: Toxic epidermal necrolysis
C: staphylococcal scalded skin syndrome
D: Psoriasis
Correct Ans:D
Explanation
The Nikolsky phenomenon is positive when the epidermis is dislodged from the dermis by
lateral, shearing pressure with a finger, resulting in an erosion. It is an important
diagnostic sign in acantholytic disorders such as pemphigus or the staphylococcal scalded
skin (SSS) syndrome or other blistering or epidermonecrotic disorders, such as toxic
epidermal necrolysis.
Ref: (2013). Introduction. In Wolff K, Johnson R, Saavedra A.P. (Eds), Fitzpatrick’s Color
Atlas and Synopsis of Clinical Dermatology, 7e.
Sample Previous Year Question on Pemphigus vulgaris based on previous Year
Questions of NEET PG, USMLE,PLAB,FMGE (MCI Screening). Please visit
www.medicoapps.org for more such Quizzes
A female presented with a skin lesions all over the body which shows positive 'bulla spread
sign'. It is characteristic of:
A: Herpes gestationis
B: Bullous pemphigoid
C: Pemphigus vulgaris
D: Herpes simplex
Correct Ans:C
Explanation
Pemphigus vulgaris is characterized by cutaneous and mucosal blisters. On application of
tangential pressure on normal skin results in formation of new bulla (Nikolsky sign) or if
applied to pre-existing bulla results in the spread of bulla (bulla spread sign). Cutaneous
lesions predominantly present on face, trunk, axillae, groins, and scalp.
Ref: Illustrated Synopsis Of Dermatology & Sexually Transmitted Diseases By Khanna, 3rd
edition, Page 64.
Sample Previous Year Question on Pemphigus vulgaris based on previous Year
Questions of NEET PG, USMLE,PLAB,FMGE (MCI Screening). Please visit
www.medicoapps.org for more such Quizzes
All of the following conditions are associated with thymoma except?
A: Myasthenia gravis
B: Pemphigus foliaceus
C: Pancytopenia
D: Hypergamma globulinemia
Correct Ans:D
Explanation
Thymoma is associated with B cell deficiency and hypogamma globulinemia. Other conditions
associated with thymoma are myasthenia gravis, autoimmune hemolytic or aplastic anemia,
pancytopenia, thrombotic thrombocytopenic purpura, pemphigus foliaceus, sjogren's syndrome,
polymyositis.
Sample Previous Year Question on Pemphigus vulgaris based on previous Year
Questions of NEET PG, USMLE,PLAB,FMGE (MCI Screening). Please visit
www.medicoapps.org for more such Quizzes
In which of the following systemic condition, conjunctival xerosis is seen as a clinical
finding?
A: Vitamin-A deficiency
B: Trachoma
C: Pemphigus
D: All of the above
Correct Ans:D
Explanation
Depending upon the etiology, conjunctival xerosis can be
divided into two groups, parenchymatous and epithelial
xerosis.
 Parenchymatous xerosis occurs following
cicatricial disorganization of the conjunctiva due
to local causes which can be in the form of,
o Widespread destructive interstitial conjunctivitis
as seen in trachoma, diphtheritic membranous
conjunctivitis, Stevens-Johnson syndrome,
pemphigus or pemphigoid conjunctivitis,
thermal, chemical or radiational burns of
conjunctiva.
o Exposure to conjunctiva to air as seen in marked
degree of proptosis, facial palsy, ectropion, lack
of blinking (as in coma), and lagophthalmos due
to symblepharon.
 Epithelial xerosis occurs due to hypovitaminosis-
A.
Ref: Ophthalmology By Khurana, 4th edition, Page
84.
Sample Previous Year Question on Pemphigus vulgaris based on previous Year
Questions of NEET PG, USMLE,PLAB,FMGE (MCI Screening). Please visit
www.medicoapps.org for more such Quizzes

Más contenido relacionado

La actualidad más candente

Erythema multiforme, Steven-Johnson syndrome and Toxic Epidermal Necrolysis
Erythema multiforme, Steven-Johnson syndrome and Toxic Epidermal NecrolysisErythema multiforme, Steven-Johnson syndrome and Toxic Epidermal Necrolysis
Erythema multiforme, Steven-Johnson syndrome and Toxic Epidermal NecrolysisBinaya Subedi
 
vesiculobullous lesions, pempigus ppt
vesiculobullous lesions, pempigus  pptvesiculobullous lesions, pempigus  ppt
vesiculobullous lesions, pempigus pptmadhusudhan reddy
 
MCQ August with answers - Dr Ameen Alawadhi
MCQ August with answers - Dr Ameen AlawadhiMCQ August with answers - Dr Ameen Alawadhi
MCQ August with answers - Dr Ameen Alawadhiaskadermatologist
 
ETAS_MCQ_12 bullous diseases and blistering
ETAS_MCQ_12 bullous diseases and blisteringETAS_MCQ_12 bullous diseases and blistering
ETAS_MCQ_12 bullous diseases and blisteringDerma202
 
Cutaneous mucinoses
Cutaneous mucinosesCutaneous mucinoses
Cutaneous mucinosesheera sanju
 
ETAS_MCQ_09 pediatric dermatology
ETAS_MCQ_09 pediatric dermatologyETAS_MCQ_09 pediatric dermatology
ETAS_MCQ_09 pediatric dermatologyDerma202
 
Vesiculobullous diseases
Vesiculobullous diseasesVesiculobullous diseases
Vesiculobullous diseasesAshish Soni
 
NECROTISING FASCIITIS- The flesh eating infection
NECROTISING FASCIITIS- The flesh eating infectionNECROTISING FASCIITIS- The flesh eating infection
NECROTISING FASCIITIS- The flesh eating infectionSelvaraj Balasubramani
 
ETAS_MCQ_05 dermatopathology
ETAS_MCQ_05 dermatopathologyETAS_MCQ_05 dermatopathology
ETAS_MCQ_05 dermatopathologyDerma202
 
ETAS_MCQ_08 infectious diseases of the skin
ETAS_MCQ_08 infectious diseases of the skinETAS_MCQ_08 infectious diseases of the skin
ETAS_MCQ_08 infectious diseases of the skinDerma202
 
MCQ July with answers - Dr Ameen Alawadhi
MCQ July with answers - Dr Ameen AlawadhiMCQ July with answers - Dr Ameen Alawadhi
MCQ July with answers - Dr Ameen Alawadhiaskadermatologist
 
Immunofluorescence in dermatopathology
Immunofluorescence in dermatopathologyImmunofluorescence in dermatopathology
Immunofluorescence in dermatopathologySumit Mandal
 
Dermatology board review
Dermatology board reviewDermatology board review
Dermatology board reviewAhmed Amer
 
Basal cell carcinoma presented by Dr.Varughese.
Basal cell carcinoma presented by Dr.Varughese.Basal cell carcinoma presented by Dr.Varughese.
Basal cell carcinoma presented by Dr.Varughese.Dr. Varughese George
 

La actualidad más candente (20)

Erythema multiforme, Steven-Johnson syndrome and Toxic Epidermal Necrolysis
Erythema multiforme, Steven-Johnson syndrome and Toxic Epidermal NecrolysisErythema multiforme, Steven-Johnson syndrome and Toxic Epidermal Necrolysis
Erythema multiforme, Steven-Johnson syndrome and Toxic Epidermal Necrolysis
 
Tzanck Smear
Tzanck SmearTzanck Smear
Tzanck Smear
 
vesiculobullous lesions, pempigus ppt
vesiculobullous lesions, pempigus  pptvesiculobullous lesions, pempigus  ppt
vesiculobullous lesions, pempigus ppt
 
MCQ August with answers - Dr Ameen Alawadhi
MCQ August with answers - Dr Ameen AlawadhiMCQ August with answers - Dr Ameen Alawadhi
MCQ August with answers - Dr Ameen Alawadhi
 
ETAS_MCQ_12 bullous diseases and blistering
ETAS_MCQ_12 bullous diseases and blisteringETAS_MCQ_12 bullous diseases and blistering
ETAS_MCQ_12 bullous diseases and blistering
 
Cutaneous mucinoses
Cutaneous mucinosesCutaneous mucinoses
Cutaneous mucinoses
 
ETAS_MCQ_09 pediatric dermatology
ETAS_MCQ_09 pediatric dermatologyETAS_MCQ_09 pediatric dermatology
ETAS_MCQ_09 pediatric dermatology
 
Vesiculobullous diseases
Vesiculobullous diseasesVesiculobullous diseases
Vesiculobullous diseases
 
Nikolsky sign
Nikolsky signNikolsky sign
Nikolsky sign
 
Lichen planus
Lichen planusLichen planus
Lichen planus
 
NECROTISING FASCIITIS- The flesh eating infection
NECROTISING FASCIITIS- The flesh eating infectionNECROTISING FASCIITIS- The flesh eating infection
NECROTISING FASCIITIS- The flesh eating infection
 
ETAS_MCQ_05 dermatopathology
ETAS_MCQ_05 dermatopathologyETAS_MCQ_05 dermatopathology
ETAS_MCQ_05 dermatopathology
 
Angioedema
AngioedemaAngioedema
Angioedema
 
Necrotizing fasciitis
Necrotizing  fasciitisNecrotizing  fasciitis
Necrotizing fasciitis
 
ETAS_MCQ_08 infectious diseases of the skin
ETAS_MCQ_08 infectious diseases of the skinETAS_MCQ_08 infectious diseases of the skin
ETAS_MCQ_08 infectious diseases of the skin
 
Behcet's disease
 Behcet's disease Behcet's disease
Behcet's disease
 
MCQ July with answers - Dr Ameen Alawadhi
MCQ July with answers - Dr Ameen AlawadhiMCQ July with answers - Dr Ameen Alawadhi
MCQ July with answers - Dr Ameen Alawadhi
 
Immunofluorescence in dermatopathology
Immunofluorescence in dermatopathologyImmunofluorescence in dermatopathology
Immunofluorescence in dermatopathology
 
Dermatology board review
Dermatology board reviewDermatology board review
Dermatology board review
 
Basal cell carcinoma presented by Dr.Varughese.
Basal cell carcinoma presented by Dr.Varughese.Basal cell carcinoma presented by Dr.Varughese.
Basal cell carcinoma presented by Dr.Varughese.
 

Similar a Previous year question on pemphigus vulgaris based on neet pg, usmle, plab and fmge or mci screening exams

Previous year question on staining based on neet pg, usmle, plab and fmge or ...
Previous year question on staining based on neet pg, usmle, plab and fmge or ...Previous year question on staining based on neet pg, usmle, plab and fmge or ...
Previous year question on staining based on neet pg, usmle, plab and fmge or ...Abhishek Gupta
 
Deramatology MRCGP Qs
Deramatology MRCGP QsDeramatology MRCGP Qs
Deramatology MRCGP Qsssnsharifa
 
Common Skin Diseases
Common Skin DiseasesCommon Skin Diseases
Common Skin Diseasesdoctorshazly
 
CASE REPORT ON PEMPHIGUS FOLIACEUS
CASE REPORT ON PEMPHIGUS FOLIACEUSCASE REPORT ON PEMPHIGUS FOLIACEUS
CASE REPORT ON PEMPHIGUS FOLIACEUSkrishna mathiyarasan
 
Autoimmune bullous dermatoses
Autoimmune bullous dermatosesAutoimmune bullous dermatoses
Autoimmune bullous dermatosesHabrol Afzam
 
Bullous diseases(group a)
Bullous diseases(group a)Bullous diseases(group a)
Bullous diseases(group a)Habrol Afzam
 
mnemonics in dermatology
mnemonics in dermatologymnemonics in dermatology
mnemonics in dermatologyAyman Ayman
 
Pemphigus - intra epidermal blistering disorders
Pemphigus - intra epidermal blistering disordersPemphigus - intra epidermal blistering disorders
Pemphigus - intra epidermal blistering disordersPreethi
 
oral pemphigus vulgaris effect on systemic health
oral pemphigus vulgaris effect on systemic healthoral pemphigus vulgaris effect on systemic health
oral pemphigus vulgaris effect on systemic healthPriyanka Pai
 
Auto immune disorders of the oral cavity
Auto immune disorders of the oral cavityAuto immune disorders of the oral cavity
Auto immune disorders of the oral cavityPrashanth Ramachandra
 
ORAL VESICULOBULLOUS LESION
ORAL VESICULOBULLOUS LESIONORAL VESICULOBULLOUS LESION
ORAL VESICULOBULLOUS LESIONSHREYADAS49
 
Toxic Epidermal Necrolysis Probably Due to Cosmetic Cream
Toxic Epidermal Necrolysis Probably Due to Cosmetic CreamToxic Epidermal Necrolysis Probably Due to Cosmetic Cream
Toxic Epidermal Necrolysis Probably Due to Cosmetic Creamv2zq
 
Ulcerative lesions/prosthodontic courses
Ulcerative lesions/prosthodontic coursesUlcerative lesions/prosthodontic courses
Ulcerative lesions/prosthodontic coursesIndian dental academy
 
Extramammary Paget’s disease a review of the literature.pdf
Extramammary Paget’s disease a review of the literature.pdfExtramammary Paget’s disease a review of the literature.pdf
Extramammary Paget’s disease a review of the literature.pdfAnna443020
 

Similar a Previous year question on pemphigus vulgaris based on neet pg, usmle, plab and fmge or mci screening exams (20)

Previous year question on staining based on neet pg, usmle, plab and fmge or ...
Previous year question on staining based on neet pg, usmle, plab and fmge or ...Previous year question on staining based on neet pg, usmle, plab and fmge or ...
Previous year question on staining based on neet pg, usmle, plab and fmge or ...
 
Deramatology MRCGP Qs
Deramatology MRCGP QsDeramatology MRCGP Qs
Deramatology MRCGP Qs
 
Dermatology 5th year, 3rd lecture (Dr. Kazhan)
Dermatology 5th year, 3rd lecture (Dr. Kazhan)Dermatology 5th year, 3rd lecture (Dr. Kazhan)
Dermatology 5th year, 3rd lecture (Dr. Kazhan)
 
Common Skin Diseases
Common Skin DiseasesCommon Skin Diseases
Common Skin Diseases
 
CASE REPORT ON PEMPHIGUS FOLIACEUS
CASE REPORT ON PEMPHIGUS FOLIACEUSCASE REPORT ON PEMPHIGUS FOLIACEUS
CASE REPORT ON PEMPHIGUS FOLIACEUS
 
vesiculobullous lesions
vesiculobullous lesionsvesiculobullous lesions
vesiculobullous lesions
 
Open Journal of Pediatrics & Neonatal Care
Open Journal of Pediatrics & Neonatal CareOpen Journal of Pediatrics & Neonatal Care
Open Journal of Pediatrics & Neonatal Care
 
Skin and Soft Tissue Infections
Skin and Soft Tissue InfectionsSkin and Soft Tissue Infections
Skin and Soft Tissue Infections
 
Oral pemphigus vulgaris
Oral pemphigus vulgaris Oral pemphigus vulgaris
Oral pemphigus vulgaris
 
Autoimmune bullous dermatoses
Autoimmune bullous dermatosesAutoimmune bullous dermatoses
Autoimmune bullous dermatoses
 
Bullous diseases(group a)
Bullous diseases(group a)Bullous diseases(group a)
Bullous diseases(group a)
 
mnemonics in dermatology
mnemonics in dermatologymnemonics in dermatology
mnemonics in dermatology
 
Pemphigus - intra epidermal blistering disorders
Pemphigus - intra epidermal blistering disordersPemphigus - intra epidermal blistering disorders
Pemphigus - intra epidermal blistering disorders
 
oral pemphigus vulgaris effect on systemic health
oral pemphigus vulgaris effect on systemic healthoral pemphigus vulgaris effect on systemic health
oral pemphigus vulgaris effect on systemic health
 
Auto immune disorders of the oral cavity
Auto immune disorders of the oral cavityAuto immune disorders of the oral cavity
Auto immune disorders of the oral cavity
 
ORAL VESICULOBULLOUS LESION
ORAL VESICULOBULLOUS LESIONORAL VESICULOBULLOUS LESION
ORAL VESICULOBULLOUS LESION
 
Desquamative Gingivitis
Desquamative GingivitisDesquamative Gingivitis
Desquamative Gingivitis
 
Toxic Epidermal Necrolysis Probably Due to Cosmetic Cream
Toxic Epidermal Necrolysis Probably Due to Cosmetic CreamToxic Epidermal Necrolysis Probably Due to Cosmetic Cream
Toxic Epidermal Necrolysis Probably Due to Cosmetic Cream
 
Ulcerative lesions/prosthodontic courses
Ulcerative lesions/prosthodontic coursesUlcerative lesions/prosthodontic courses
Ulcerative lesions/prosthodontic courses
 
Extramammary Paget’s disease a review of the literature.pdf
Extramammary Paget’s disease a review of the literature.pdfExtramammary Paget’s disease a review of the literature.pdf
Extramammary Paget’s disease a review of the literature.pdf
 

Más de Medico Apps

Previous year question on hepatomegaly based on neet pg, usmle, plab and fmge...
Previous year question on hepatomegaly based on neet pg, usmle, plab and fmge...Previous year question on hepatomegaly based on neet pg, usmle, plab and fmge...
Previous year question on hepatomegaly based on neet pg, usmle, plab and fmge...Medico Apps
 
Previous year question on rabies based on neet pg, usmle, plab and fmge or mc...
Previous year question on rabies based on neet pg, usmle, plab and fmge or mc...Previous year question on rabies based on neet pg, usmle, plab and fmge or mc...
Previous year question on rabies based on neet pg, usmle, plab and fmge or mc...Medico Apps
 
Previous year question on medulloblastoma based on neet pg, usmle, plab and f...
Previous year question on medulloblastoma based on neet pg, usmle, plab and f...Previous year question on medulloblastoma based on neet pg, usmle, plab and f...
Previous year question on medulloblastoma based on neet pg, usmle, plab and f...Medico Apps
 
Previous year question on cataract based on neet pg, usmle, plab and fmge or ...
Previous year question on cataract based on neet pg, usmle, plab and fmge or ...Previous year question on cataract based on neet pg, usmle, plab and fmge or ...
Previous year question on cataract based on neet pg, usmle, plab and fmge or ...Medico Apps
 
Previous year question on pharyngeal arches embryology based on neet pg, usml...
Previous year question on pharyngeal arches embryology based on neet pg, usml...Previous year question on pharyngeal arches embryology based on neet pg, usml...
Previous year question on pharyngeal arches embryology based on neet pg, usml...Medico Apps
 
Previous year question on leptospirosis based on neet pg, usmle, plab and fmg...
Previous year question on leptospirosis based on neet pg, usmle, plab and fmg...Previous year question on leptospirosis based on neet pg, usmle, plab and fmg...
Previous year question on leptospirosis based on neet pg, usmle, plab and fmg...Medico Apps
 
Previous year question on glycolysis based on neet pg, usmle, plab and fmge o...
Previous year question on glycolysis based on neet pg, usmle, plab and fmge o...Previous year question on glycolysis based on neet pg, usmle, plab and fmge o...
Previous year question on glycolysis based on neet pg, usmle, plab and fmge o...Medico Apps
 
Previous year question on glycolysis based on neet pg, usmle, plab and fmge o...
Previous year question on glycolysis based on neet pg, usmle, plab and fmge o...Previous year question on glycolysis based on neet pg, usmle, plab and fmge o...
Previous year question on glycolysis based on neet pg, usmle, plab and fmge o...Medico Apps
 
Previous year question on bone cyst based on neet pg, usmle, plab and fmge or...
Previous year question on bone cyst based on neet pg, usmle, plab and fmge or...Previous year question on bone cyst based on neet pg, usmle, plab and fmge or...
Previous year question on bone cyst based on neet pg, usmle, plab and fmge or...Medico Apps
 
Previous year question on ketamine based on neet pg, usmle, plab and fmge or ...
Previous year question on ketamine based on neet pg, usmle, plab and fmge or ...Previous year question on ketamine based on neet pg, usmle, plab and fmge or ...
Previous year question on ketamine based on neet pg, usmle, plab and fmge or ...Medico Apps
 
Ketamine (anaesthesia )sample questions based on neet pg , usmle, plab and fm...
Ketamine (anaesthesia )sample questions based on neet pg , usmle, plab and fm...Ketamine (anaesthesia )sample questions based on neet pg , usmle, plab and fm...
Ketamine (anaesthesia )sample questions based on neet pg , usmle, plab and fm...Medico Apps
 
Radiology most important signs sample questions based on neet pg , usmle, pla...
Radiology most important signs sample questions based on neet pg , usmle, pla...Radiology most important signs sample questions based on neet pg , usmle, pla...
Radiology most important signs sample questions based on neet pg , usmle, pla...Medico Apps
 
Hiv aids sample questions based on neet pg , usmle, plab and fmge pattern (mc...
Hiv aids sample questions based on neet pg , usmle, plab and fmge pattern (mc...Hiv aids sample questions based on neet pg , usmle, plab and fmge pattern (mc...
Hiv aids sample questions based on neet pg , usmle, plab and fmge pattern (mc...Medico Apps
 
Hiv aids sample questions based on neet pg , usmle, plab and fmge pattern (mc...
Hiv aids sample questions based on neet pg , usmle, plab and fmge pattern (mc...Hiv aids sample questions based on neet pg , usmle, plab and fmge pattern (mc...
Hiv aids sample questions based on neet pg , usmle, plab and fmge pattern (mc...Medico Apps
 
Hiv aids sample questions based on neet pg , usmle, plab and fmge pattern (mc...
Hiv aids sample questions based on neet pg , usmle, plab and fmge pattern (mc...Hiv aids sample questions based on neet pg , usmle, plab and fmge pattern (mc...
Hiv aids sample questions based on neet pg , usmle, plab and fmge pattern (mc...Medico Apps
 
Hiv aids sample questions based on neet pg , usmle, plab and fmge pattern (mc...
Hiv aids sample questions based on neet pg , usmle, plab and fmge pattern (mc...Hiv aids sample questions based on neet pg , usmle, plab and fmge pattern (mc...
Hiv aids sample questions based on neet pg , usmle, plab and fmge pattern (mc...Medico Apps
 
Hiv aids sample questions based on neet pg , usmle, plab and fmge pattern (mc...
Hiv aids sample questions based on neet pg , usmle, plab and fmge pattern (mc...Hiv aids sample questions based on neet pg , usmle, plab and fmge pattern (mc...
Hiv aids sample questions based on neet pg , usmle, plab and fmge pattern (mc...Medico Apps
 
Meconium stained baby sample questions based on neet pg , usmle, plab and fmg...
Meconium stained baby sample questions based on neet pg , usmle, plab and fmg...Meconium stained baby sample questions based on neet pg , usmle, plab and fmg...
Meconium stained baby sample questions based on neet pg , usmle, plab and fmg...Medico Apps
 
Pancreatitis sample questions based on neet pg , usmle, plab and fmge pattern...
Pancreatitis sample questions based on neet pg , usmle, plab and fmge pattern...Pancreatitis sample questions based on neet pg , usmle, plab and fmge pattern...
Pancreatitis sample questions based on neet pg , usmle, plab and fmge pattern...Medico Apps
 
Alzheimers disease sample questions based on neet pg , usmle, plab and fmge p...
Alzheimers disease sample questions based on neet pg , usmle, plab and fmge p...Alzheimers disease sample questions based on neet pg , usmle, plab and fmge p...
Alzheimers disease sample questions based on neet pg , usmle, plab and fmge p...Medico Apps
 

Más de Medico Apps (20)

Previous year question on hepatomegaly based on neet pg, usmle, plab and fmge...
Previous year question on hepatomegaly based on neet pg, usmle, plab and fmge...Previous year question on hepatomegaly based on neet pg, usmle, plab and fmge...
Previous year question on hepatomegaly based on neet pg, usmle, plab and fmge...
 
Previous year question on rabies based on neet pg, usmle, plab and fmge or mc...
Previous year question on rabies based on neet pg, usmle, plab and fmge or mc...Previous year question on rabies based on neet pg, usmle, plab and fmge or mc...
Previous year question on rabies based on neet pg, usmle, plab and fmge or mc...
 
Previous year question on medulloblastoma based on neet pg, usmle, plab and f...
Previous year question on medulloblastoma based on neet pg, usmle, plab and f...Previous year question on medulloblastoma based on neet pg, usmle, plab and f...
Previous year question on medulloblastoma based on neet pg, usmle, plab and f...
 
Previous year question on cataract based on neet pg, usmle, plab and fmge or ...
Previous year question on cataract based on neet pg, usmle, plab and fmge or ...Previous year question on cataract based on neet pg, usmle, plab and fmge or ...
Previous year question on cataract based on neet pg, usmle, plab and fmge or ...
 
Previous year question on pharyngeal arches embryology based on neet pg, usml...
Previous year question on pharyngeal arches embryology based on neet pg, usml...Previous year question on pharyngeal arches embryology based on neet pg, usml...
Previous year question on pharyngeal arches embryology based on neet pg, usml...
 
Previous year question on leptospirosis based on neet pg, usmle, plab and fmg...
Previous year question on leptospirosis based on neet pg, usmle, plab and fmg...Previous year question on leptospirosis based on neet pg, usmle, plab and fmg...
Previous year question on leptospirosis based on neet pg, usmle, plab and fmg...
 
Previous year question on glycolysis based on neet pg, usmle, plab and fmge o...
Previous year question on glycolysis based on neet pg, usmle, plab and fmge o...Previous year question on glycolysis based on neet pg, usmle, plab and fmge o...
Previous year question on glycolysis based on neet pg, usmle, plab and fmge o...
 
Previous year question on glycolysis based on neet pg, usmle, plab and fmge o...
Previous year question on glycolysis based on neet pg, usmle, plab and fmge o...Previous year question on glycolysis based on neet pg, usmle, plab and fmge o...
Previous year question on glycolysis based on neet pg, usmle, plab and fmge o...
 
Previous year question on bone cyst based on neet pg, usmle, plab and fmge or...
Previous year question on bone cyst based on neet pg, usmle, plab and fmge or...Previous year question on bone cyst based on neet pg, usmle, plab and fmge or...
Previous year question on bone cyst based on neet pg, usmle, plab and fmge or...
 
Previous year question on ketamine based on neet pg, usmle, plab and fmge or ...
Previous year question on ketamine based on neet pg, usmle, plab and fmge or ...Previous year question on ketamine based on neet pg, usmle, plab and fmge or ...
Previous year question on ketamine based on neet pg, usmle, plab and fmge or ...
 
Ketamine (anaesthesia )sample questions based on neet pg , usmle, plab and fm...
Ketamine (anaesthesia )sample questions based on neet pg , usmle, plab and fm...Ketamine (anaesthesia )sample questions based on neet pg , usmle, plab and fm...
Ketamine (anaesthesia )sample questions based on neet pg , usmle, plab and fm...
 
Radiology most important signs sample questions based on neet pg , usmle, pla...
Radiology most important signs sample questions based on neet pg , usmle, pla...Radiology most important signs sample questions based on neet pg , usmle, pla...
Radiology most important signs sample questions based on neet pg , usmle, pla...
 
Hiv aids sample questions based on neet pg , usmle, plab and fmge pattern (mc...
Hiv aids sample questions based on neet pg , usmle, plab and fmge pattern (mc...Hiv aids sample questions based on neet pg , usmle, plab and fmge pattern (mc...
Hiv aids sample questions based on neet pg , usmle, plab and fmge pattern (mc...
 
Hiv aids sample questions based on neet pg , usmle, plab and fmge pattern (mc...
Hiv aids sample questions based on neet pg , usmle, plab and fmge pattern (mc...Hiv aids sample questions based on neet pg , usmle, plab and fmge pattern (mc...
Hiv aids sample questions based on neet pg , usmle, plab and fmge pattern (mc...
 
Hiv aids sample questions based on neet pg , usmle, plab and fmge pattern (mc...
Hiv aids sample questions based on neet pg , usmle, plab and fmge pattern (mc...Hiv aids sample questions based on neet pg , usmle, plab and fmge pattern (mc...
Hiv aids sample questions based on neet pg , usmle, plab and fmge pattern (mc...
 
Hiv aids sample questions based on neet pg , usmle, plab and fmge pattern (mc...
Hiv aids sample questions based on neet pg , usmle, plab and fmge pattern (mc...Hiv aids sample questions based on neet pg , usmle, plab and fmge pattern (mc...
Hiv aids sample questions based on neet pg , usmle, plab and fmge pattern (mc...
 
Hiv aids sample questions based on neet pg , usmle, plab and fmge pattern (mc...
Hiv aids sample questions based on neet pg , usmle, plab and fmge pattern (mc...Hiv aids sample questions based on neet pg , usmle, plab and fmge pattern (mc...
Hiv aids sample questions based on neet pg , usmle, plab and fmge pattern (mc...
 
Meconium stained baby sample questions based on neet pg , usmle, plab and fmg...
Meconium stained baby sample questions based on neet pg , usmle, plab and fmg...Meconium stained baby sample questions based on neet pg , usmle, plab and fmg...
Meconium stained baby sample questions based on neet pg , usmle, plab and fmg...
 
Pancreatitis sample questions based on neet pg , usmle, plab and fmge pattern...
Pancreatitis sample questions based on neet pg , usmle, plab and fmge pattern...Pancreatitis sample questions based on neet pg , usmle, plab and fmge pattern...
Pancreatitis sample questions based on neet pg , usmle, plab and fmge pattern...
 
Alzheimers disease sample questions based on neet pg , usmle, plab and fmge p...
Alzheimers disease sample questions based on neet pg , usmle, plab and fmge p...Alzheimers disease sample questions based on neet pg , usmle, plab and fmge p...
Alzheimers disease sample questions based on neet pg , usmle, plab and fmge p...
 

Último

Call Girls Bareilly Just Call 8250077686 Top Class Call Girl Service Available
Call Girls Bareilly Just Call 8250077686 Top Class Call Girl Service AvailableCall Girls Bareilly Just Call 8250077686 Top Class Call Girl Service Available
Call Girls Bareilly Just Call 8250077686 Top Class Call Girl Service AvailableDipal Arora
 
Call Girls Siliguri Just Call 8250077686 Top Class Call Girl Service Available
Call Girls Siliguri Just Call 8250077686 Top Class Call Girl Service AvailableCall Girls Siliguri Just Call 8250077686 Top Class Call Girl Service Available
Call Girls Siliguri Just Call 8250077686 Top Class Call Girl Service AvailableDipal Arora
 
Best Rate (Guwahati ) Call Girls Guwahati ⟟ 8617370543 ⟟ High Class Call Girl...
Best Rate (Guwahati ) Call Girls Guwahati ⟟ 8617370543 ⟟ High Class Call Girl...Best Rate (Guwahati ) Call Girls Guwahati ⟟ 8617370543 ⟟ High Class Call Girl...
Best Rate (Guwahati ) Call Girls Guwahati ⟟ 8617370543 ⟟ High Class Call Girl...Dipal Arora
 
VIP Hyderabad Call Girls Bahadurpally 7877925207 ₹5000 To 25K With AC Room 💚😋
VIP Hyderabad Call Girls Bahadurpally 7877925207 ₹5000 To 25K With AC Room 💚😋VIP Hyderabad Call Girls Bahadurpally 7877925207 ₹5000 To 25K With AC Room 💚😋
VIP Hyderabad Call Girls Bahadurpally 7877925207 ₹5000 To 25K With AC Room 💚😋TANUJA PANDEY
 
Call Girls Ooty Just Call 8250077686 Top Class Call Girl Service Available
Call Girls Ooty Just Call 8250077686 Top Class Call Girl Service AvailableCall Girls Ooty Just Call 8250077686 Top Class Call Girl Service Available
Call Girls Ooty Just Call 8250077686 Top Class Call Girl Service AvailableDipal Arora
 
VIP Service Call Girls Sindhi Colony 📳 7877925207 For 18+ VIP Call Girl At Th...
VIP Service Call Girls Sindhi Colony 📳 7877925207 For 18+ VIP Call Girl At Th...VIP Service Call Girls Sindhi Colony 📳 7877925207 For 18+ VIP Call Girl At Th...
VIP Service Call Girls Sindhi Colony 📳 7877925207 For 18+ VIP Call Girl At Th...jageshsingh5554
 
Call Girls Bangalore Just Call 8250077686 Top Class Call Girl Service Available
Call Girls Bangalore Just Call 8250077686 Top Class Call Girl Service AvailableCall Girls Bangalore Just Call 8250077686 Top Class Call Girl Service Available
Call Girls Bangalore Just Call 8250077686 Top Class Call Girl Service AvailableDipal Arora
 
Top Quality Call Girl Service Kalyanpur 6378878445 Available Call Girls Any Time
Top Quality Call Girl Service Kalyanpur 6378878445 Available Call Girls Any TimeTop Quality Call Girl Service Kalyanpur 6378878445 Available Call Girls Any Time
Top Quality Call Girl Service Kalyanpur 6378878445 Available Call Girls Any TimeCall Girls Delhi
 
Night 7k to 12k Chennai City Center Call Girls 👉👉 7427069034⭐⭐ 100% Genuine E...
Night 7k to 12k Chennai City Center Call Girls 👉👉 7427069034⭐⭐ 100% Genuine E...Night 7k to 12k Chennai City Center Call Girls 👉👉 7427069034⭐⭐ 100% Genuine E...
Night 7k to 12k Chennai City Center Call Girls 👉👉 7427069034⭐⭐ 100% Genuine E...hotbabesbook
 
Call Girls Nagpur Just Call 9907093804 Top Class Call Girl Service Available
Call Girls Nagpur Just Call 9907093804 Top Class Call Girl Service AvailableCall Girls Nagpur Just Call 9907093804 Top Class Call Girl Service Available
Call Girls Nagpur Just Call 9907093804 Top Class Call Girl Service AvailableDipal Arora
 
Top Rated Bangalore Call Girls Richmond Circle ⟟ 9332606886 ⟟ Call Me For Ge...
Top Rated Bangalore Call Girls Richmond Circle ⟟  9332606886 ⟟ Call Me For Ge...Top Rated Bangalore Call Girls Richmond Circle ⟟  9332606886 ⟟ Call Me For Ge...
Top Rated Bangalore Call Girls Richmond Circle ⟟ 9332606886 ⟟ Call Me For Ge...narwatsonia7
 
Mumbai ] (Call Girls) in Mumbai 10k @ I'm VIP Independent Escorts Girls 98333...
Mumbai ] (Call Girls) in Mumbai 10k @ I'm VIP Independent Escorts Girls 98333...Mumbai ] (Call Girls) in Mumbai 10k @ I'm VIP Independent Escorts Girls 98333...
Mumbai ] (Call Girls) in Mumbai 10k @ I'm VIP Independent Escorts Girls 98333...Ishani Gupta
 
All Time Service Available Call Girls Marine Drive 📳 9820252231 For 18+ VIP C...
All Time Service Available Call Girls Marine Drive 📳 9820252231 For 18+ VIP C...All Time Service Available Call Girls Marine Drive 📳 9820252231 For 18+ VIP C...
All Time Service Available Call Girls Marine Drive 📳 9820252231 For 18+ VIP C...Arohi Goyal
 
Book Paid Powai Call Girls Mumbai 𖠋 9930245274 𖠋Low Budget Full Independent H...
Book Paid Powai Call Girls Mumbai 𖠋 9930245274 𖠋Low Budget Full Independent H...Book Paid Powai Call Girls Mumbai 𖠋 9930245274 𖠋Low Budget Full Independent H...
Book Paid Powai Call Girls Mumbai 𖠋 9930245274 𖠋Low Budget Full Independent H...Call Girls in Nagpur High Profile
 
Premium Bangalore Call Girls Jigani Dail 6378878445 Escort Service For Hot Ma...
Premium Bangalore Call Girls Jigani Dail 6378878445 Escort Service For Hot Ma...Premium Bangalore Call Girls Jigani Dail 6378878445 Escort Service For Hot Ma...
Premium Bangalore Call Girls Jigani Dail 6378878445 Escort Service For Hot Ma...tanya dube
 
Call Girls Tirupati Just Call 8250077686 Top Class Call Girl Service Available
Call Girls Tirupati Just Call 8250077686 Top Class Call Girl Service AvailableCall Girls Tirupati Just Call 8250077686 Top Class Call Girl Service Available
Call Girls Tirupati Just Call 8250077686 Top Class Call Girl Service AvailableDipal Arora
 
Call Girls Visakhapatnam Just Call 8250077686 Top Class Call Girl Service Ava...
Call Girls Visakhapatnam Just Call 8250077686 Top Class Call Girl Service Ava...Call Girls Visakhapatnam Just Call 8250077686 Top Class Call Girl Service Ava...
Call Girls Visakhapatnam Just Call 8250077686 Top Class Call Girl Service Ava...Dipal Arora
 
Call Girls Coimbatore Just Call 9907093804 Top Class Call Girl Service Available
Call Girls Coimbatore Just Call 9907093804 Top Class Call Girl Service AvailableCall Girls Coimbatore Just Call 9907093804 Top Class Call Girl Service Available
Call Girls Coimbatore Just Call 9907093804 Top Class Call Girl Service AvailableDipal Arora
 
Call Girls Bhubaneswar Just Call 9907093804 Top Class Call Girl Service Avail...
Call Girls Bhubaneswar Just Call 9907093804 Top Class Call Girl Service Avail...Call Girls Bhubaneswar Just Call 9907093804 Top Class Call Girl Service Avail...
Call Girls Bhubaneswar Just Call 9907093804 Top Class Call Girl Service Avail...Dipal Arora
 
Top Rated Bangalore Call Girls Mg Road ⟟ 9332606886 ⟟ Call Me For Genuine S...
Top Rated Bangalore Call Girls Mg Road ⟟   9332606886 ⟟ Call Me For Genuine S...Top Rated Bangalore Call Girls Mg Road ⟟   9332606886 ⟟ Call Me For Genuine S...
Top Rated Bangalore Call Girls Mg Road ⟟ 9332606886 ⟟ Call Me For Genuine S...narwatsonia7
 

Último (20)

Call Girls Bareilly Just Call 8250077686 Top Class Call Girl Service Available
Call Girls Bareilly Just Call 8250077686 Top Class Call Girl Service AvailableCall Girls Bareilly Just Call 8250077686 Top Class Call Girl Service Available
Call Girls Bareilly Just Call 8250077686 Top Class Call Girl Service Available
 
Call Girls Siliguri Just Call 8250077686 Top Class Call Girl Service Available
Call Girls Siliguri Just Call 8250077686 Top Class Call Girl Service AvailableCall Girls Siliguri Just Call 8250077686 Top Class Call Girl Service Available
Call Girls Siliguri Just Call 8250077686 Top Class Call Girl Service Available
 
Best Rate (Guwahati ) Call Girls Guwahati ⟟ 8617370543 ⟟ High Class Call Girl...
Best Rate (Guwahati ) Call Girls Guwahati ⟟ 8617370543 ⟟ High Class Call Girl...Best Rate (Guwahati ) Call Girls Guwahati ⟟ 8617370543 ⟟ High Class Call Girl...
Best Rate (Guwahati ) Call Girls Guwahati ⟟ 8617370543 ⟟ High Class Call Girl...
 
VIP Hyderabad Call Girls Bahadurpally 7877925207 ₹5000 To 25K With AC Room 💚😋
VIP Hyderabad Call Girls Bahadurpally 7877925207 ₹5000 To 25K With AC Room 💚😋VIP Hyderabad Call Girls Bahadurpally 7877925207 ₹5000 To 25K With AC Room 💚😋
VIP Hyderabad Call Girls Bahadurpally 7877925207 ₹5000 To 25K With AC Room 💚😋
 
Call Girls Ooty Just Call 8250077686 Top Class Call Girl Service Available
Call Girls Ooty Just Call 8250077686 Top Class Call Girl Service AvailableCall Girls Ooty Just Call 8250077686 Top Class Call Girl Service Available
Call Girls Ooty Just Call 8250077686 Top Class Call Girl Service Available
 
VIP Service Call Girls Sindhi Colony 📳 7877925207 For 18+ VIP Call Girl At Th...
VIP Service Call Girls Sindhi Colony 📳 7877925207 For 18+ VIP Call Girl At Th...VIP Service Call Girls Sindhi Colony 📳 7877925207 For 18+ VIP Call Girl At Th...
VIP Service Call Girls Sindhi Colony 📳 7877925207 For 18+ VIP Call Girl At Th...
 
Call Girls Bangalore Just Call 8250077686 Top Class Call Girl Service Available
Call Girls Bangalore Just Call 8250077686 Top Class Call Girl Service AvailableCall Girls Bangalore Just Call 8250077686 Top Class Call Girl Service Available
Call Girls Bangalore Just Call 8250077686 Top Class Call Girl Service Available
 
Top Quality Call Girl Service Kalyanpur 6378878445 Available Call Girls Any Time
Top Quality Call Girl Service Kalyanpur 6378878445 Available Call Girls Any TimeTop Quality Call Girl Service Kalyanpur 6378878445 Available Call Girls Any Time
Top Quality Call Girl Service Kalyanpur 6378878445 Available Call Girls Any Time
 
Night 7k to 12k Chennai City Center Call Girls 👉👉 7427069034⭐⭐ 100% Genuine E...
Night 7k to 12k Chennai City Center Call Girls 👉👉 7427069034⭐⭐ 100% Genuine E...Night 7k to 12k Chennai City Center Call Girls 👉👉 7427069034⭐⭐ 100% Genuine E...
Night 7k to 12k Chennai City Center Call Girls 👉👉 7427069034⭐⭐ 100% Genuine E...
 
Call Girls Nagpur Just Call 9907093804 Top Class Call Girl Service Available
Call Girls Nagpur Just Call 9907093804 Top Class Call Girl Service AvailableCall Girls Nagpur Just Call 9907093804 Top Class Call Girl Service Available
Call Girls Nagpur Just Call 9907093804 Top Class Call Girl Service Available
 
Top Rated Bangalore Call Girls Richmond Circle ⟟ 9332606886 ⟟ Call Me For Ge...
Top Rated Bangalore Call Girls Richmond Circle ⟟  9332606886 ⟟ Call Me For Ge...Top Rated Bangalore Call Girls Richmond Circle ⟟  9332606886 ⟟ Call Me For Ge...
Top Rated Bangalore Call Girls Richmond Circle ⟟ 9332606886 ⟟ Call Me For Ge...
 
Mumbai ] (Call Girls) in Mumbai 10k @ I'm VIP Independent Escorts Girls 98333...
Mumbai ] (Call Girls) in Mumbai 10k @ I'm VIP Independent Escorts Girls 98333...Mumbai ] (Call Girls) in Mumbai 10k @ I'm VIP Independent Escorts Girls 98333...
Mumbai ] (Call Girls) in Mumbai 10k @ I'm VIP Independent Escorts Girls 98333...
 
All Time Service Available Call Girls Marine Drive 📳 9820252231 For 18+ VIP C...
All Time Service Available Call Girls Marine Drive 📳 9820252231 For 18+ VIP C...All Time Service Available Call Girls Marine Drive 📳 9820252231 For 18+ VIP C...
All Time Service Available Call Girls Marine Drive 📳 9820252231 For 18+ VIP C...
 
Book Paid Powai Call Girls Mumbai 𖠋 9930245274 𖠋Low Budget Full Independent H...
Book Paid Powai Call Girls Mumbai 𖠋 9930245274 𖠋Low Budget Full Independent H...Book Paid Powai Call Girls Mumbai 𖠋 9930245274 𖠋Low Budget Full Independent H...
Book Paid Powai Call Girls Mumbai 𖠋 9930245274 𖠋Low Budget Full Independent H...
 
Premium Bangalore Call Girls Jigani Dail 6378878445 Escort Service For Hot Ma...
Premium Bangalore Call Girls Jigani Dail 6378878445 Escort Service For Hot Ma...Premium Bangalore Call Girls Jigani Dail 6378878445 Escort Service For Hot Ma...
Premium Bangalore Call Girls Jigani Dail 6378878445 Escort Service For Hot Ma...
 
Call Girls Tirupati Just Call 8250077686 Top Class Call Girl Service Available
Call Girls Tirupati Just Call 8250077686 Top Class Call Girl Service AvailableCall Girls Tirupati Just Call 8250077686 Top Class Call Girl Service Available
Call Girls Tirupati Just Call 8250077686 Top Class Call Girl Service Available
 
Call Girls Visakhapatnam Just Call 8250077686 Top Class Call Girl Service Ava...
Call Girls Visakhapatnam Just Call 8250077686 Top Class Call Girl Service Ava...Call Girls Visakhapatnam Just Call 8250077686 Top Class Call Girl Service Ava...
Call Girls Visakhapatnam Just Call 8250077686 Top Class Call Girl Service Ava...
 
Call Girls Coimbatore Just Call 9907093804 Top Class Call Girl Service Available
Call Girls Coimbatore Just Call 9907093804 Top Class Call Girl Service AvailableCall Girls Coimbatore Just Call 9907093804 Top Class Call Girl Service Available
Call Girls Coimbatore Just Call 9907093804 Top Class Call Girl Service Available
 
Call Girls Bhubaneswar Just Call 9907093804 Top Class Call Girl Service Avail...
Call Girls Bhubaneswar Just Call 9907093804 Top Class Call Girl Service Avail...Call Girls Bhubaneswar Just Call 9907093804 Top Class Call Girl Service Avail...
Call Girls Bhubaneswar Just Call 9907093804 Top Class Call Girl Service Avail...
 
Top Rated Bangalore Call Girls Mg Road ⟟ 9332606886 ⟟ Call Me For Genuine S...
Top Rated Bangalore Call Girls Mg Road ⟟   9332606886 ⟟ Call Me For Genuine S...Top Rated Bangalore Call Girls Mg Road ⟟   9332606886 ⟟ Call Me For Genuine S...
Top Rated Bangalore Call Girls Mg Road ⟟ 9332606886 ⟟ Call Me For Genuine S...
 

Previous year question on pemphigus vulgaris based on neet pg, usmle, plab and fmge or mci screening exams

  • 1. A 27-year-old sexually active male develops a vesiculobullous lesion on the glans soon after taking tablet paracetamol for fever. The lesion healed with hyperpigmentation. The most likely diagnosis is: A: Behcet's syndrome B: Herpes genitalis C: Fixed drug eruption D: Pemphigus vulgaris Correct Ans:C Explanation An adverse cutaneous reaction to an ingested drug with characteristic clinical features is fixed drug eruption. Presentation with a solitary lesion in the genital skin that heals with hyperpigmentation. Soon after ingestion of paracetamol, one of the most commonly implicated agents for FDE, leads to the diagnosis of FDE. Ref: Anthony Du Vivier, Phillip H. McKee, Chapter 17, “Reactive Disorder of The Skin and Drug Eruptions”, In the book, “Atlas of Clinical Dermatology”, Elsivier Publication, 2002, 3rd Edition, Spain, Page 367 Sample Previous Year Question on Pemphigus vulgaris based on previous Year Questions of NEET PG, USMLE,PLAB,FMGE (MCI Screening). Please visit www.medicoapps.org for more such Quizzes A 22 year old female presents with dirty warty papules in the seborrheic areas especially in the summer. Her father also gives a h/o similar lesions. What is the most probable diagnosis? A: Pemphigus foliaceus B: Keratosis pilaris C: Darier’s disease D: Seborrheic dermatitis Correct Ans:C Explanation It is also known as Keratosis follicularis with autosomal dominant inheritance.
  • 2. Sample Previous Year Question on Pemphigus vulgaris based on previous Year Questions of NEET PG, USMLE,PLAB,FMGE (MCI Screening). Please visit www.medicoapps.org for more such Quizzes The “dilapidated brick wall” appearance is classically seen in the histopathology of the following? A: Hailey Hailey disease B: Pemphigus vulgaris C: Darier's disease D: Bullous pemphigoid Correct Ans:A Explanation Hailey Hailey disease/familial benign chronic pemphigus has full thickness partial acantholysis resulting in this appearance. Ref: Rook Textbook of Dermatology, 8th Edition, Page 39.33. Sample Previous Year Question on Pemphigus vulgaris based on previous Year Questions of NEET PG, USMLE,PLAB,FMGE (MCI Screening). Please visit www.medicoapps.org for more such Quizzes Koebner's phenomenon is seen in all of the following conditions, EXCEPT: A: Lichen planus B: Ichthyosi s C: Psoriasis D: Pemphigus Correct Ans:B Explanation Koebner’s phenomenon refers to development of morphologically identical lesions in the traumatized uninvolved skin of the patients who have cutaneous diseases. It is also known as isomorphic phenomenon. Conditions associated with Koebner’s phenomenon are:  Warts
  • 3.  Molluscum contagiosum  Autoimmune (vitiligo)  Psoriasis  Pemphigus  Lichen planus Ref: An Aid to the MRCP: Essential Lists, Facts and Mnemonics By Nicholas Boeckx, page 184. Sample Previous Year Question on Pemphigus vulgaris based on previous Year Questions of NEET PG, USMLE,PLAB,FMGE (MCI Screening). Please visit www.medicoapps.org for more such Quizzes A 30 year old pregnant woman presents to a physician with painful oral ulcers. Physical examination demonstrates widespread erosions of her mucous membranes. Close examination reveals a friable mucosa, but no well-defined aphthous ulcers. Biopsy of perilesional mucosa demonstrates acantholysis; direct immunofluorescence demonstrates an intraepidermal band of IgG and C3. Which of the following is the most likely diagnosis? A: Bullous pemphigoid B: Dermatitis herpetiformis C: Herpes simplex I D: Pemphigus vulgaris Correct Ans:D Explanation This is pemphigus vulgaris, in which autoantibody directed against transmembrane cadherin adhesion molecules induced acantholysis (breakdown of epithelial cell-cell connections) with resulting intraepidermal blister formation. It may develop spontaneously or following triggers such as drugs (thiols, penicillamine), physical injury (burns), cancer, pregnancy, other skin diseases, and emotional stress. Pemphigus vulgaris is a relatively rare blistering disease; it is seen more commonly in patients with Jewish or Mediterranean heritage. In addition to the usually prominent oral ulcers, uncomfortable skin erosions can also occur when the blisters rupture rapidly and are not observed. The epidermis at the edge of these erosions is often easily disrupted by sliding pressure (Nikolsky sign). Bullous pemphigoid is characterized by deeper blisters, occurring at the dermal-epidermal junction. Dermatitis herpetiformis is characterized by severe, intense pruritus and groups of papules and vesicles. Herpes simplex I or II can show multinucleated giant cells on scrapings of the ulcer base. Ref: Suurmond D. (2009). Section 34. Disorders of the Mouth. In D. Suurmond (Ed), Fitzpatrick's Color Atlas & Synopsis of Clinical Dermatology, 6e.
  • 4. Sample Previous Year Question on Pemphigus vulgaris based on previous Year Questions of NEET PG, USMLE,PLAB,FMGE (MCI Screening). Please visit www.medicoapps.org for more such Quizzes Tzanck preparation is used for the following skin conditions, EXCEPT: A: Pemphigus B: SSS syndrome C: Senear Usher syndrome D: Fungal infections Correct Ans:D Explanation Microscopic examination of cells obtained from the base of vesicles (Tzanck preparation) may reveal the presence of acantholytic cells in the acantholytic diseases (e.g., pemphigus or SSS syndrome) or of giant epithelial cells and multinucleated giant cells (containing 10– 12 nuclei) in herpes simplex, herpes zoster, and varicella. Material from the base of a vesicle obtained by gentle curettage with a scalpel is smeared on a glass slide, stained with either Giemsa or Wright stain or methylene blue, and examined to determine whether there are acantholytic or giant epithelial cells, which are diagnostic. Pemphigus erythematosus is also known as Senear-Usher syndrome. Ref: (2013). Introduction. In Wolff K, Johnson R, Saavedra A.P. (Eds), Fitzpatrick’s Color Atlas and Synopsis of Clinical Dermatology, 7e. Sample Previous Year Question on Pemphigus vulgaris based on previous Year Questions of NEET PG, USMLE,PLAB,FMGE (MCI Screening). Please visit www.medicoapps.org for more such Quizzes A 30 year old male presents with itchy papulo-vesicular lesions on the extremities, knees, elbows and buttocks since one year. Direct immunofluorescence staining of the lesions showed IgA deposition at dermoepidermal junction. Which of the following represents the most probable diagnosis? A: Pemphigus vulgaris B: Bullous pemphigoid C: Dermatitis herpetiformis D: Nummular eczema Correct Ans:C Explanation
  • 5. Patient is showing features of dermatitis herpetiformis. Dermatitis herpetiformis is a chronic, intensely itchy blistering disease characterized by the presence of papules and vesicles occurring predominantly on the extensor surfaces of the body such as the elbows, knees, buttocks and scapula. It is seen in association with gluten sensitive enteropathy. Diagnosis is made by the presence of IgA deposits in the dermoepidermal junction by direct immunoflouresence examination of a perilesional or normal appearing skin biopsy sample. Treatment includes dapsone and gluten free diet. Ref: NORD Guide to Rare Disorders: National Organization for Rare Disorders, Page 103; Atlas of Clinical Dermatology By Anthony Du Vivier, 3rd Edition, Pages 418, 426; Dermatology in Clinical Practice By S. W. Lanigan, Page 170 Sample Previous Year Question on Pemphigus vulgaris based on previous Year Questions of NEET PG, USMLE,PLAB,FMGE (MCI Screening). Please visit www.medicoapps.org for more such Quizzes A 40 year old male developed persistant oral ulcers followed by multiple flaccid bullae on trunk and extremities. Direct immunofluorescence examination of a skin biopsy showed intercellular IgG deposits in the epidermis. The most probable diagnosis is: A: Pemphigus vulgaris B: Bullous Pemphigoid C: Bullous Lupus erythematosus D: Epidermolysis bullosa acquisita Correct Ans:A Explanation It is an autoimmune intraepidermal blistering disease due to antibodies against desmoglein 3. It begins as painful oral erosions followed by flaccid blisters on the trunk, face, scalp. Ref: Current Literature Dermatology By Pasricha, Pages 197, 202; Rook’s Textbook of Dermatology, 8th Edition. Sample Previous Year Question on Pemphigus vulgaris based on previous Year Questions of NEET PG, USMLE,PLAB,FMGE (MCI Screening). Please visit www.medicoapps.org for more such Quizzes Which of the following disorder is associated with acantholysis? A: Pemphigoid B: Pemphigus vulgaris C: Erythema multiforme
  • 6. D: Dermatitis herpetiformis Correct Ans:B Explanation Acantholysis refers to loss of cohesion between epidermal cells as a result of destruction of intercellular substance. Acantholytic cells are round cells with large hyperchromatic nuclei and perinuclear halo. The primary lesion of pemphigus vulgaris is acantholysis in the epidermis. Its histologic hallmark is acantholysis which is suprabasal, and the basal cells remain attached to the basement membrane. Ref: Tropical Dermatopathology By Singh, Page 121; Pathology of The Head and Neck By Antonio Cardesa, Page 74. Sample Previous Year Question on Pemphigus vulgaris based on previous Year Questions of NEET PG, USMLE,PLAB,FMGE (MCI Screening). Please visit www.medicoapps.org for more such Quizzes A 45 yr old female developed recurrent oral erosions followed by multiple flaccid bullae on trunk and extremities. A tzanck smear showed acantholytic cells and direct immunofluoroscence showed intercellular IgG deposits in the epidermis. The most probable diagnosis is: A: Bullous Pemphigoid B: Stevens Johnson syndrome C: Herpes simplex 1 infection D: Pemphigus vulgaris Correct Ans:D Explanation Recurrent, painful oral lesions and cutaneous lesions in the form of flaccid bullae are characteristic of phemphigus vulgaris. Positive Tzanck smear, IgG deposits on direct immunofluoroscence positive nikolsky’s sign are features of phemphigus vulgaris. Ref: Textbook of Dermatology, Venereology and Leprology By Devinder Mohan Thappa, 2nd Edition, Chapter 12, Pages 152-53; Dermatology (A coloured Hand Textbook) By R. J. G. Rycroft, S. J. Robertson, Sarah H. Wakeli, 2nd Edition, Part 1, Pages 54, 55 Sample Previous Year Question on Pemphigus vulgaris based on previous Year Questions of NEET PG, USMLE,PLAB,FMGE (MCI Screening). Please visit www.medicoapps.org for more such Quizzes
  • 7. Itching associated with linear IgA deposition in dermal papillae is a feature of: A: Bullous disease of childhood B: Lichenoid bullous disease C: Dermatitis herpetiformis D: Pemphigus vulgaris Correct Ans:C Explanation The cutaneous lesions of Dermatitis herpetiformis (DH) in childhood resemble those in adult and are severely pruritic papulovesicles. DH is associated with granular deposition of IgA antibody within the dermal papillae. Ref: Cambridge Textbook of Accident and Emergency Medicine edited by David V. Skinner, Andrew Swain, Colin Robertson, J. W. Rodney Peyton, 1997, Page 1051 Sample Previous Year Question on Pemphigus vulgaris based on previous Year Questions of NEET PG, USMLE,PLAB,FMGE (MCI Screening). Please visit www.medicoapps.org for more such Quizzes Which of the following drug is not associated with drug induced pemphigus? A: Rifampicin B: Penicilli n C: Captopril D: Furosemide Correct Ans:D Explanation Penicillin, cephalosporins, penicillamine, rifampicin, captopril, piroxicam, phenylbutazone are drugs commonly associated with drug induced pemphigus. Sample Previous Year Question on Pemphigus vulgaris based on previous Year Questions of NEET PG, USMLE,PLAB,FMGE (MCI Screening). Please visit www.medicoapps.org for more such Quizzes Pemphigus vulgaris is characterized by all, except:
  • 8. A: Positive Nikolsky’s sign B: Oral erosions C: Subepidermal bulla D: Tzanck smear showing acantholytic cells Correct Ans:C Explanation Pemphigus vulgaris is an intraepidermal blistering disease. Sample Previous Year Question on Pemphigus vulgaris based on previous Year Questions of NEET PG, USMLE,PLAB,FMGE (MCI Screening). Please visit www.medicoapps.org for more such Quizzes Scenario: A 35 year old lady is diagnosed with Pemphigus Vulgaris by her dermatologist. Assertion: Pemphigus vulgaris presents with flaccid blisters and oral erosions in a middle aged person. Reason: It is a intraepidermal blistering disorder due to antibodies against desmogleins. A: Both Assertion and Reason are true, and Reason is the correct explanation for Assertion B: Both Assertion and Reason are true, and Reason is not the correct explanation for Assertion C: Assertion is true, but Reason is false D: Assertion is false, but Reason is true Correct Ans:A Explanation Pemphigus vulgaris is an intraepidermal blistering disease due to antibodies against desmogleins (epidermal attachment complexes). It presents with flaccid blisters and oral erosions in a middle aged person. The subepidermal blistering disorders present with tense blisters. Ref: Rook’s Textbook of Dermatology, 8th Edition, Page 40.3 Sample Previous Year Question on Pemphigus vulgaris based on previous Year Questions of NEET PG, USMLE,PLAB,FMGE (MCI Screening). Please visit www.medicoapps.org for more such Quizzes
  • 9. A person presents with hemorrhagic fluid in a tense blister at dermoepidermal junction. Most probable diagnosis is: A: Pemphigoid B: Pemphigus vulgaris C: Pemphigus vegetans D: Drug induced pemphigus Correct Ans:A Explanation The pemphigoid group includes a series of chronic diseases in which blisters form within the lamina lucida of the basement membrane dermoepidermal junction). The blister roof is thus the entire epidermis so it is stable. Frequently the blisters are hemorrhagic, fluid filled and tense. No accantholysis occurs, so the Tzanck test is negative. Antibodies are directed against various components of the basement membrane zone. Characteristic of pemphigus vulgaris is an intradermal blister occurring immediately above the basal cell layer and associated with acantholytic cells. The blisters rapidly ruptures, leaving non healing, painful erosions. Pemphigus vegetans usually presents clinically as serpiginous ulcers. Microscopically, the epithelium tends to proliferate and become verruciform. Acantholytic cells may not be conspicuous and eosinophil microabscesses are the most typical histological feature. Drug induced pemphigus can present as pemphigus foliaceous, pemphigus erythematosus, or as pemphigus vulgaris. Moist erythematous crusted scaling plaques and superficial friable blisters are seen. Common drugs responsible are thiol compounds, drugs having a sulph-hydryl group, penicillamine. Sample Previous Year Question on Pemphigus vulgaris based on previous Year Questions of NEET PG, USMLE,PLAB,FMGE (MCI Screening). Please visit www.medicoapps.org for more such Quizzes Fogo selvagem is a type of: A: Pemphigus vulgaris B: Pemghigus vegetans C: Pemphigus foliaceus D: Bullous pemphigoid Correct Ans:C Explanation
  • 10. Endemic pemphigus foliaceus common in rural parts of South America, particularly certain states of Brazil is known as fogo selvagem (wild fire) caused by bite of black fly. Ref: Rook's textbook of dermatology, 8th edition, Pg 40.13 Sample Previous Year Question on Pemphigus vulgaris based on previous Year Questions of NEET PG, USMLE,PLAB,FMGE (MCI Screening). Please visit www.medicoapps.org for more such Quizzes FTU is a measure of: A: Area involved in severe drug reactions B: Amount of topical drug to be applied C: Mucosal involvement in pemphigus D: Concentration of drug in topical preparations Correct Ans:B Explanation An approximate practical measure of topical medication is the fingertip unit, the quantity of ointment, extruded from a tube with a nozzle of 5 mm diameter extending from the distal crease of the forefinger to ventral aspect of the fingertip. This unit weighs approximately 0.49 g in males and 0.43 g in females and covers, on average, an area of approx 300 cm2. Sample Previous Year Question on Pemphigus vulgaris based on previous Year Questions of NEET PG, USMLE,PLAB,FMGE (MCI Screening). Please visit www.medicoapps.org for more such Quizzes Which of the following presents with a "string of pearl appearance"? A: Pemphigus vulgaris B: Bullous pemphigoid C: Linear IgA disease D: Dermatitis herpetiformis Correct Ans:C Explanation
  • 11. The lesions in Linear IgA disease with a bimodal age distribution ie <5yrs & > 60 yrs comprise of urticated plaques and papules, and annular, polycyclic lesions often with blistering around the edge, the string of pearls sign/cluster of jewels sign. Sample Previous Year Question on Pemphigus vulgaris based on previous Year Questions of NEET PG, USMLE,PLAB,FMGE (MCI Screening). Please visit www.medicoapps.org for more such Quizzes A biopsy of affected skin in Pemphigus vulgaris would show which of the following? A: Acantholysis B: Balloon degeneration C: Reticular changes D: Spongiosis Correct Ans:A Explanation The pathologic changes in pemphigus vulgaris are acantholysis, clefts, and blister formation in the intraepidermal areas. Acantholysis is the separation of keratinocytes from one another. The loss of cohesion or contact between cells begins with the detachment of tonofilaments from desmosomes. Evidence indicates that an IgG autoantibody actually induces these changes. Good to know:  Balloon degeneration and reticular changes are both histologic markers of herpes simplex viral-induced lesions.  Spongiosis is a general term referring to serous exudates between cells of the epidermis, with an inflammatory infiltrate in the dermis. This is frequently seen indermatitis lesions. Ref:Longo D.L., Fauci A.S., Kasper D.L., Hauser S.L., Jameson J.L., Loscalzo J. (2012). Chapter 54. Immunologically Mediated Skin Diseases. In D.L. Longo, A.S. Fauci, D.L. Kasper, S.L. Hauser, J.L. Jameson, J. Loscalzo (Eds), Harrison's Principles of Internal Medicine, 18e. Sample Previous Year Question on Pemphigus vulgaris based on previous Year Questions of NEET PG, USMLE,PLAB,FMGE (MCI Screening). Please visit www.medicoapps.org for more such Quizzes Which of the following tests would be MOST useful for confirmation of Pemphigus vulgaris?
  • 12. A: Bacterial culture B: Complete blood count C: Fungal culture D: Tissue biopsy with direct immunofluorescence Correct Ans:D Explanation Direct immunofluorescence (DIF) is of great value in the early diagnosis of pemphigus vulgaris. DIF shows intercellular IgG throughout the epidermis or the oral epithelium. IgG is found in both involved and clinically normal skin in nearly all patients with pemphigus. In acantholytic areas, C3 deposition is also reliably found. DIF results remain positive for a long time and may still be positive many years after clinical remission.  Bacterial culture is helpful in superinfected lesions, but does not help in confirming a diagnosis of pemphigus.  Complete blood count is helpful in infectious processes.  Fungal culture is used in tinea lesions. Ref: Longo D.L., Fauci A.S., Kasper D.L., Hauser S.L., Jameson J.L., Loscalzo J. (2012). Chapter 54. Immunologically Mediated Skin Diseases. In D.L. Longo, A.S. Fauci, D.L. Kasper, S.L. Hauser, J.L. Jameson, J. Loscalzo (Eds), Harrison's Principles of Internal Medicine, 18e. Sample Previous Year Question on Pemphigus vulgaris based on previous Year Questions of NEET PG, USMLE,PLAB,FMGE (MCI Screening). Please visit www.medicoapps.org for more such Quizzes Patient with gluten-sensitive enteropathy has a lifelong history of periodic crops of intensely pruritic, grouped, papular or vesicular lesions on the elbows, knees, sacrum, and shoulders. Because the vesicles are intensely pruritic, the patient routinely scratches the top off them, which relieves the pruritus. Which of the following is the most likely diagnosis? A: Bullous pemphigoid B: Dermatitis herpetiformis C: Herpes simplex I D: Pemphigus vulgaris Correct Ans:B Explanation The condition described is dermatitis herpetiformis, which is strongly associated with gluten-sensitive enteropathy (celiac sprue), and often has a life-long, intermittent course. Clinically, patients have (excoriated) groups of papules and vesicles on an erythematous base. The lesions tend to involve the extensor surfaces of the extremities and the buttocks. Microscopically, the lesions show subepidermal papillary dermal neutrophilic abscesses,
  • 13. with granular deposits of IgA and C3 in dermal papillary tips. Patients may respond dramatically to dapsone therapy. Bullous pemphigoid produces large, tense blisters. Herpes simplex I is characterized by crops of vesicles, but is not particularly associated with gluten-sensitive enteropathy. Pemphigus vulgaris usually shows prominent oral involvement. Ref: DeWitt C.A., Buescher L.S., Stone S.P. (2012). Chapter 153. Cutaneous Manifestations of Internal Malignant Disease: Cutaneous Paraneoplastic Syndromes. In L.A. Goldsmith, S.I. Katz, B.A. Gilchrest, A.S. Paller, D.J. Leffell, N.A. Dallas (Eds), Fitzpatrick's Dermatology in General Medicine, 8e. Sample Previous Year Question on Pemphigus vulgaris based on previous Year Questions of NEET PG, USMLE,PLAB,FMGE (MCI Screening). Please visit www.medicoapps.org for more such Quizzes 'Bulla spread sign' is seen in which of the following condition? A: Herpes gestationalis B: Bullous pemphigoid C: Pemphigus vulgaris D: Herpes simplex Correct Ans:C Explanation Pemphigus vulgaris is associated with positive Nikolsky sign and Asboe- Hansen sign or Bullae spread sign. In positive bullae spread sign, pressing the top of existing bullae will lead to spread of bullae to adjacent normal appearing skin. Positive Nikolsky sign refers to shearing of the epidermis by exerting lateral pressure on the normal skin adjacent to an active blister. Pemphigus vulgaris is an acquired chronic disease in which blisters develop on normal appearing skin and mucous membrane. In this condition the pemphigus antibodies attach to extracellular domain of desmoglein and interfere with its attachment to similar domains on other cells reducing cell cell adherence. It is characterised by the development of flaccid clear blister which rupture and evolve into an inflamed erosion. Ref: Blistering Skin Diseases By Lawrence S. Chan page 26. Sample Previous Year Question on Pemphigus vulgaris based on previous Year Questions of NEET PG, USMLE,PLAB,FMGE (MCI Screening). Please visit www.medicoapps.org for more such Quizzes A 30 year old male presented with itchy, papulovesicular lesions in the extensor aspect of elbows and knees. Biopsy of the lesion showed dermoepidermal blister with micro-
  • 14. abscesses. Immunofluorescence showed deposits of IgA in the tips of dermal papillae. What is the MOST likely diagnosis? A: Bullous Impetigo B: Bullous pemphigoid C: Pemphigus vulgaris D: Dermatitis herpetiformis Correct Ans:D Explanation sensitive enteropathy. In this condition absorption of gluten induces the formation of circulating immune complexes which deposit in the dermal papillae causing inflammation and dermoepidermal split. Skin lesions are in the form of extremely itchy grouped papulovesicular lesions that develop on normal or erythematous skin. The most frequently involve the extensors(elbow, knee, buttocks, shoulder and sacral area). Biopsy shows: dermoepidermal blister with collection of polymorphs at the tip of dermal papillae.Immunofluorescence showed deposits of IgA in the tips of dermal papillae. Serology: Anti endomysial and anti reticulin antibodies are present in most patients. Ref: Illustrated Synopsis Of Dermatology & Sexually Transmitted Diseases (3Nd By Khanna page 71. Sample Previous Year Question on Pemphigus vulgaris based on previous Year Questions of NEET PG, USMLE,PLAB,FMGE (MCI Screening). Please visit www.medicoapps.org for more such Quizzes Intraepidermal bullae formation occur in which of the following condition? A: Bullous Impetigo B: Bullous pemphigoid C: Pemphigus vulgaris D: Dermatitis herpetiformis Correct Ans:C Explanation Pemphigus vulgaris is an intraepidermal bullous disorder where the split is suprabasal. Pemphigus is an autoimmune disorder characterised by the production of IgG autoantibodies against desmoglein which are involved in intercellular adhesion. These antibodies induce keratinocytes to release serine proteases which dissolve the intercellular substance. The site of split in 4 main variants of pemphigus are:
  • 15.  Pemphigus vulgaris: split is suprabasal  Pemphigus vegetans: split is suprabasal  Pemphigus foliaceus: blister is either in the granular layer or just below the horny layer  Pemphigus erythematosus: blister is either in the granular layer or just below stratum corneum Ref: Illustrated Synopsis Of Dermatology & Sexually Transmitted Diseases By Khanna page 64. Sample Previous Year Question on Pemphigus vulgaris based on previous Year Questions of NEET PG, USMLE,PLAB,FMGE (MCI Screening). Please visit www.medicoapps.org for more such Quizzes A 30 year old male presents to the dermatologist with silvery, scaling plaques on his elbows and knees. His mother has been afflicted with the same condition in the past. The most likely diagnosis is ? A: Acne rosacea B: Acne vulgaris C: Pemphigus vulgaris D: Psoriasis vulgaris Correct Ans:D Explanation Psoriasis vulgaris usually appears on the nails, knees, elbows, and scalp. It does not generally affect the mucous membranes. Lesions are well-demarcated coral-colored plaques with a white or silvery scale (classic clue). Histologically, epidermal hyperplasia causing thickening and lengthening of the rete ridges is apparent, as is thinning of the epidermis over the dermal papillae. There is a recognized genetic component to this condition. Peak incidence is at age 30. Here's a point-saving strategy: It is easy to get confused on this question because of the similar looking answer choices, especially under time pressure. Be extra careful in such cases to mark the proper choice on your answer grid. Acne rosacea affects the central face. Erythema, telangiectasias, acneform lesions (papules, cysts, pustules), and rhinophyma (telangiectasias and hyperplasia of nasal soft tissue) are found in various combinations. It is common from ages 30-50. Women are affected three times more frequently than men, but the syndrome is more severe in the latter. Acne vulgaris causes comedones, papules, and cysts. It may be related to hormones, drugs, diet, irritants, and genetic factors. Allergy to Propionibacterium acnes has been found to contribute to this condition. Pemphigus vulgaris starts with small vesicles, usually on the oral or nasal mucosa, then spreads to other parts of the body. Bullae are delicate and flaccid. The condition is due to autoantibodies to intercellular junctions between keratinocytes. Nikolsky's sign (production
  • 16. of blistering by light stroking or rubbing of the skin) is positive. Pemphigus is most common from ages 40-60. Ref: Suurmond D. (2009). Section 3. Psoriasis. In D. Suurmond (Ed), Fitzpatrick's Color Atlas & Synopsis of Clinical Dermatology, 6e. Sample Previous Year Question on Pemphigus vulgaris based on previous Year Questions of NEET PG, USMLE,PLAB,FMGE (MCI Screening). Please visit www.medicoapps.org for more such Quizzes A 60 year old male presents to the dermatologist with a complaint of tender blisters on his arm and flank. Physical examination reveals blisters and flaccid bullae; a few have ruptured, leaving red, sore, denuded areas. Which of the following findings would suggest the diagnosis of pemphigus vulgaris as opposed to bullous pemphigoid? A: Eosinophils within bullae B: IgA deposits on basement membrane C: Negative Nikolsky sign D: Oral mucosal lesions Correct Ans:D Explanation Pemphigus vulgaris is associated with chronic, severe bullae formation on the skin and oral mucosa. Mucosal lesions are extremely rare in bullous pemphigoid. This can be used clinically to guide therapy, although a skin biopsy should be taken to confirm the diagnosis. Both diseases are characterized by formation of tender bullae that can rupture,leaving red, raw areas. Pemphigus vulgaris patients eventually became febrile and lose weight, and if untreated, most will die within one year. Bullous pemphigoid lesions tend to heal and the patients do very well. This prognostic difference is an important distinction in dermatological medicine. Eosinophils within blisters provide an important clue supporting bullous pemphigoid as the diagnosis which must be ascertained with histologic examination. The vesicles in pemphigus vulgaris mostly contain rounded acantholytic keratinocytes "floating" within. IgA deposits seen with immunofluorescence on the basement membrane is characteristic of a third bullae-forming disease known as dermatitis herpetiformis. Vesicles form between the dermal-epidermal junction. The disease is associated with gluten sensitivity and celiac disease. IgG autoantibody activity is common to both pemphigus vulgaris and bullous pemphigoid. Ref: Payne A.S., Stanley J.R. (2012). Chapter 54. Pemphigus. In K. Wolff (Ed),Fitzpatrick's Dermatology in General Medicine, 8e.
  • 17. Sample Previous Year Question on Pemphigus vulgaris based on previous Year Questions of NEET PG, USMLE,PLAB,FMGE (MCI Screening). Please visit www.medicoapps.org for more such Quizzes The day after playing in the high grasses of a neighbor's fields, an 8 year old boy develops a weeping, vesicular, erythematous, and itchy rash on his arms, legs, and neck. Which of the following best describes this inflammatory response? A: Erythema nodosum B: Pemphigu s C: Psoriasis D: Spongiotic dermatitis Correct Ans:D Explanation Spongiotic dermatitis is seen in cases of contact dermatitis, such as poison ivy exposure. The accumulation of inflammatory cells in the superficial dermis causes marked edema, which splays epidermal keratinocytes apart and giving a spongy appearance to intercellular bridges. Grossly, the skin has a weepy appearance with frequent blistering. Erythema nodosum is a form of panniculitis, which is chronic inflammation in the subcutaneous fat lobules. Erythema nodosum presents as painful erythematous nodules, often with fever and malaise. It is associated with infections and drug reactions, and is not a contact dermatitis. Pemphigus is a genetic blistering disorder due to the production of antibodies to the intercellular cement substances in skin and mucous membranes. Psoriasis is a common chronic inflammatory disease causing plaques and scales, typically on elbows, knees, and scalp. The pathogenesis of psoriasis is still unclear; it may be a complement-mediated autoimmune process. Ref: McCalmont T.H. (2010). Chapter 8. Diseases of the Skin. In S.J. McPhee, G.D. Hammer (Eds), Pathophysiology of Disease, 6e. Sample Previous Year Question on Pemphigus vulgaris based on previous Year Questions of NEET PG, USMLE,PLAB,FMGE (MCI Screening). Please visit www.medicoapps.org for more such Quizzes A patient presents with scarring Alopecia, thinned nails, hypopigmented macular lesions over trunk and oral mucosa. The diagnosis is : A: Psoriasis B: Lepros
  • 18. y C: Lichen planus D: Pemphigus Correct Ans:C Explanation Lichen planus (LP) is a papulosquamous disorder that may affect the skin, scalp, nails, and mucous membranes. The primary cutaneous lesions are pruritic, polygonal, flat-topped, violaceous papules. Close examination of the surface of these papules often reveals a network of gray lines (Wickham's striae). The skin lesions may occur anywhere but have a predilection for the wrists, shins, lower back, and genitalia. Involvement of the scalp, lichen planopilaris, may lead to scarring alopecia, and nail involvement may lead to permanent deformity or loss of fingernails and toenails. LP commonly involves mucous membranes, particularly the buccal mucosa, where it can present a spectrum of disease from a mild, white, reticulate eruption of the mucosa, to a severe, erosive stomatitis. Erosive stomatitis may persist for years and may be linked to an increased risk of oral squamous cell carcinoma. Ref: Harrison's Internal Medicine;Part 2. Cardinal Manifestations and Presentation of Diseases ;Section 9. Alterations in the Skin ;Chapter 53. Eczema, Psoriasis, Cutaneous Infections, Acne, and Other Common Skin Disorders-Eczema and Dermatitis Sample Previous Year Question on Pemphigus vulgaris based on previous Year Questions of NEET PG, USMLE,PLAB,FMGE (MCI Screening). Please visit www.medicoapps.org for more such Quizzes A patient presents with scarring Alopecia, thinned nails, hypopigmented macular lesions over trunk and oral mucosa. The diagnosis is: A: Psoriasis B: Lepros y C: Lichen planus D: Pemphigus Correct Ans:C Explanation Lichen planus (LP) is a papulosquamous disorder that may affect the skin, scalp, nails, and mucous membranes. The primary cutaneous lesions are pruritic, polygonal, flat-topped, violaceous papules. Close examination of the surface of these papules often reveals a network of gray lines (Wickham's striae). The skin lesions may occur anywhere but have a predilection for the wrists, shins, lower back, and genitalia. Involvement of the scalp, lichen planopilaris, may lead to scarring alopecia, and nail involvement may lead to permanent deformity or loss of fingernails and toenails. LP commonly involves mucous membranes, particularly the buccal mucosa, where it can present a spectrum of disease from a mild, white, reticulate eruption of the mucosa, to a severe, erosive stomatitis. Erosive stomatitis
  • 19. may persist for years and may be linked to an increased risk of oral squamous cell carcinoma. Ref: Harrison's Internal Medicine;Part 2. Cardinal Manifestations and Presentation of Diseases ;Section 9. Alterations in the Skin ;Chapter 53. Eczema, Psoriasis, Cutaneous Infections, Acne, and Other Common Skin Disorders-Eczema and Dermatitis. Sample Previous Year Question on Pemphigus vulgaris based on previous Year Questions of NEET PG, USMLE,PLAB,FMGE (MCI Screening). Please visit www.medicoapps.org for more such Quizzes 'Bulla spread sign' is seen in : A: Herpes gestational B: Bullous pemphigoid C: Pemphigus vulgaris D: Herpes simplex Correct Ans:C Explanation In the traditional "bulla spread" sign or Lutz sign, the margin of an intact bullae is first marked by a pen. Slow, careful and unidirectional pressure applied by a finger to the bulla causes peripheral extension of the bulla beyond the marked margin. The bulla thus extended has an irregular angulated border in pemphigus vulgaris, while a regular rounded border is observed in bullous pemphigoid or other subepidermal blistering disorders. Ref :Longo D.L., Fauci A.S., Kasper D.L., Hauser S.L., Jameson J.L., Loscalzo J. (2012). Chapter 54. Immunologically Mediated Skin Diseases. In D.L. Longo, A.S. Fauci, D.L. Kasper, S.L. Hauser, J.L. Jameson, J. Loscalzo (Eds), Harrison's Principles of Internal Medicine, 18e. Sample Previous Year Question on Pemphigus vulgaris based on previous Year Questions of NEET PG, USMLE,PLAB,FMGE (MCI Screening). Please visit www.medicoapps.org for more such Quizzes Berkley membrane is seen in the following cutaneous condition: A: Psoriasis B: Pemphigus C: Tinea capitis D: Pityriasis rubra Correct Ans:A Explanation
  • 20. Psoriasis is characterized by well-circumscribed, sharply demarcated erythematous papules and/or plaques. These are covered by dry, brittle, silvery or grayish white, loosely adherent, micaceous scales. The scales are disposed in lamellar fashion. Occasionally a white blanching ring is seen around the psoriatic lesions, known asWoronoff ring. On grattage, silvery white scales come off in layers. After their removal, a characteristic coherence is observed, as if one scratches on a wax candle (candle grease sign). On further grattage, a thin peel like membrane, Berkley’s membrane, is seen which comes off as a whole. On its removal, a wet surface with multiple pinpoint bleeding is revealed. This is called Auspitz sign. Ref: Textbook of Clinical Dermatology By Sehgal, 4th edition, Page 127. Sample Previous Year Question on Pemphigus vulgaris based on previous Year Questions of NEET PG, USMLE,PLAB,FMGE (MCI Screening). Please visit www.medicoapps.org for more such Quizzes Nikolsky's sign is positive in each of the following conditions, EXCEPT: A: Pemphigus B: Toxic epidermal necrolysis C: staphylococcal scalded skin syndrome D: Psoriasis Correct Ans:D Explanation The Nikolsky phenomenon is positive when the epidermis is dislodged from the dermis by lateral, shearing pressure with a finger, resulting in an erosion. It is an important diagnostic sign in acantholytic disorders such as pemphigus or the staphylococcal scalded skin (SSS) syndrome or other blistering or epidermonecrotic disorders, such as toxic epidermal necrolysis. Ref: (2013). Introduction. In Wolff K, Johnson R, Saavedra A.P. (Eds), Fitzpatrick’s Color Atlas and Synopsis of Clinical Dermatology, 7e. Sample Previous Year Question on Pemphigus vulgaris based on previous Year Questions of NEET PG, USMLE,PLAB,FMGE (MCI Screening). Please visit www.medicoapps.org for more such Quizzes A female presented with a skin lesions all over the body which shows positive 'bulla spread sign'. It is characteristic of: A: Herpes gestationis B: Bullous pemphigoid
  • 21. C: Pemphigus vulgaris D: Herpes simplex Correct Ans:C Explanation Pemphigus vulgaris is characterized by cutaneous and mucosal blisters. On application of tangential pressure on normal skin results in formation of new bulla (Nikolsky sign) or if applied to pre-existing bulla results in the spread of bulla (bulla spread sign). Cutaneous lesions predominantly present on face, trunk, axillae, groins, and scalp. Ref: Illustrated Synopsis Of Dermatology & Sexually Transmitted Diseases By Khanna, 3rd edition, Page 64. Sample Previous Year Question on Pemphigus vulgaris based on previous Year Questions of NEET PG, USMLE,PLAB,FMGE (MCI Screening). Please visit www.medicoapps.org for more such Quizzes All of the following conditions are associated with thymoma except? A: Myasthenia gravis B: Pemphigus foliaceus C: Pancytopenia D: Hypergamma globulinemia Correct Ans:D Explanation Thymoma is associated with B cell deficiency and hypogamma globulinemia. Other conditions associated with thymoma are myasthenia gravis, autoimmune hemolytic or aplastic anemia, pancytopenia, thrombotic thrombocytopenic purpura, pemphigus foliaceus, sjogren's syndrome, polymyositis. Sample Previous Year Question on Pemphigus vulgaris based on previous Year Questions of NEET PG, USMLE,PLAB,FMGE (MCI Screening). Please visit www.medicoapps.org for more such Quizzes In which of the following systemic condition, conjunctival xerosis is seen as a clinical finding? A: Vitamin-A deficiency B: Trachoma C: Pemphigus
  • 22. D: All of the above Correct Ans:D Explanation Depending upon the etiology, conjunctival xerosis can be divided into two groups, parenchymatous and epithelial xerosis.  Parenchymatous xerosis occurs following cicatricial disorganization of the conjunctiva due to local causes which can be in the form of, o Widespread destructive interstitial conjunctivitis as seen in trachoma, diphtheritic membranous conjunctivitis, Stevens-Johnson syndrome, pemphigus or pemphigoid conjunctivitis, thermal, chemical or radiational burns of conjunctiva. o Exposure to conjunctiva to air as seen in marked degree of proptosis, facial palsy, ectropion, lack of blinking (as in coma), and lagophthalmos due to symblepharon.  Epithelial xerosis occurs due to hypovitaminosis- A. Ref: Ophthalmology By Khurana, 4th edition, Page 84. Sample Previous Year Question on Pemphigus vulgaris based on previous Year Questions of NEET PG, USMLE,PLAB,FMGE (MCI Screening). Please visit www.medicoapps.org for more such Quizzes